Alternatives to LaTeX Unicorn Meta Zoo #1: Why another podcast? Announcing...

What's the difference between using dependency injection with a container and using a service locator?

Putting Ant-Man on house arrest

Will I lose my paid in full property

How to open locks without disable device?

Can you stand up from being prone using Skirmisher outside of your turn?

Did the Roman Empire have penal colonies?

What is /etc/mtab in Linux?

With indentation set to `0em`, when using a line break, there is still an indentation of a size of a space

How can I wire a 9-position switch so that each position turns on one more LED than the one before?

How to translate "red flag" into Spanish?

Do I need to protect SFP ports and optics from dust/contaminants? If so, how?

c++ diamond problem - How to call base method only once

Map material from china not allowed to leave the country

How to not starve gigantic beasts

Is a 5 watt UHF/VHF handheld considered QRP?

As an international instructor, should I openly talk about my accent?

Expansion//Explosion and Siren Stormtamer

Are these square matrices always diagonalisable?

How would I use different systems of magic when they are capable of the same effects?

What is this word supposed to be?

Is this homebrew racial feat, Stonehide, balanced?

Where did Arya get these scars?

How to avoid introduction cliches

What is the term for a person whose job is to place products on shelves in stores?



Alternatives to LaTeX



Unicorn Meta Zoo #1: Why another podcast?
Announcing the arrival of Valued Associate #679: Cesar ManaraIs Latex the only choice, is there any (emerging) alternative?Why is LaTeX so complicated?Visual comparison between LaTeX and Word output (hyphenation, typesetting, ligatures etc)Using siunitx when submitting to APSWhich typesetting programming language is most used in publishing?Future of TeX and LaTeXWhat visual features of typeset plain text are unique to LaTeX compared to InDesign, QuarkXPress and other WYSIWYG typesetting programs?If LaTeX tools implement the TeX features from scratchSetting variables in LuaTeX texconfig?LaTeX-related eventsWhy are there no alternatives to TeX, or, why is TeX still used?Is LaTeX outdated?LaTeX in Industrylatex and e-readersGet certified in TeX/LaTeXBeyond of the LaTeXLaTeX motto/sloganAlternatives to FF NexusBook Layout in Latex












174















Are there any professional alternatives to TeX/LaTeX which produce equal or even better typesetting? Maybe expensive commercial ones. The software counts if it is comparable or superior to LaTeX (with microtype) in typesetting text or math (or both). In any case, please mention both aspects in your answer.



I am only aware of ConTeXt (which is partly based on TeX).



Note that I am not looking for an alternative for my personal use, I am just asking out of curiosity.










share|improve this question




















  • 7





    Without making any comparisons, there is Adobe inDesign

    – Scott H.
    Jun 20 '13 at 19:20






  • 10





    @ScottH. -- to the best of my knowledge, indesign doesn't (yet?) handle math.

    – barbara beeton
    Jun 20 '13 at 19:26






  • 3





    @barbarabeeton: But there is a plugin named MathMagic for Adobe InDesign to handle math.

    – kiss my armpit
    Jun 20 '13 at 19:55






  • 6





    There is an interesting comparison LaTeX vs. InDesign (but a bit old): zinktypografie.nl/latex.php?lang=en

    – student
    Jun 20 '13 at 20:06






  • 4





    I tried MathMagic plugin with inDesign. It was terrible. It is just like using the equation editor with Word. Has to go through loops to finally enter an equation. One equation. That is it. After 10 minutes of using it, I uninstalled the whole thing, and also uninstalled indesign (was trial version). With no direct Latex support inside indesign, I mean direct support, indesign will never make it in science, math and engineering. Ok for magazines and articles. Nothing more.

    – Nasser
    Mar 4 '14 at 7:10
















174















Are there any professional alternatives to TeX/LaTeX which produce equal or even better typesetting? Maybe expensive commercial ones. The software counts if it is comparable or superior to LaTeX (with microtype) in typesetting text or math (or both). In any case, please mention both aspects in your answer.



I am only aware of ConTeXt (which is partly based on TeX).



Note that I am not looking for an alternative for my personal use, I am just asking out of curiosity.










share|improve this question




















  • 7





    Without making any comparisons, there is Adobe inDesign

    – Scott H.
    Jun 20 '13 at 19:20






  • 10





    @ScottH. -- to the best of my knowledge, indesign doesn't (yet?) handle math.

    – barbara beeton
    Jun 20 '13 at 19:26






  • 3





    @barbarabeeton: But there is a plugin named MathMagic for Adobe InDesign to handle math.

    – kiss my armpit
    Jun 20 '13 at 19:55






  • 6





    There is an interesting comparison LaTeX vs. InDesign (but a bit old): zinktypografie.nl/latex.php?lang=en

    – student
    Jun 20 '13 at 20:06






  • 4





    I tried MathMagic plugin with inDesign. It was terrible. It is just like using the equation editor with Word. Has to go through loops to finally enter an equation. One equation. That is it. After 10 minutes of using it, I uninstalled the whole thing, and also uninstalled indesign (was trial version). With no direct Latex support inside indesign, I mean direct support, indesign will never make it in science, math and engineering. Ok for magazines and articles. Nothing more.

    – Nasser
    Mar 4 '14 at 7:10














174












174








174


89






Are there any professional alternatives to TeX/LaTeX which produce equal or even better typesetting? Maybe expensive commercial ones. The software counts if it is comparable or superior to LaTeX (with microtype) in typesetting text or math (or both). In any case, please mention both aspects in your answer.



I am only aware of ConTeXt (which is partly based on TeX).



Note that I am not looking for an alternative for my personal use, I am just asking out of curiosity.










share|improve this question
















Are there any professional alternatives to TeX/LaTeX which produce equal or even better typesetting? Maybe expensive commercial ones. The software counts if it is comparable or superior to LaTeX (with microtype) in typesetting text or math (or both). In any case, please mention both aspects in your answer.



I am only aware of ConTeXt (which is partly based on TeX).



Note that I am not looking for an alternative for my personal use, I am just asking out of curiosity.







typography latex-misc






share|improve this question















share|improve this question













share|improve this question




share|improve this question








edited Nov 7 '15 at 10:28









David Carlisle

501k4211471897




501k4211471897










asked Jun 20 '13 at 19:18









studentstudent

12.5k24104176




12.5k24104176








  • 7





    Without making any comparisons, there is Adobe inDesign

    – Scott H.
    Jun 20 '13 at 19:20






  • 10





    @ScottH. -- to the best of my knowledge, indesign doesn't (yet?) handle math.

    – barbara beeton
    Jun 20 '13 at 19:26






  • 3





    @barbarabeeton: But there is a plugin named MathMagic for Adobe InDesign to handle math.

    – kiss my armpit
    Jun 20 '13 at 19:55






  • 6





    There is an interesting comparison LaTeX vs. InDesign (but a bit old): zinktypografie.nl/latex.php?lang=en

    – student
    Jun 20 '13 at 20:06






  • 4





    I tried MathMagic plugin with inDesign. It was terrible. It is just like using the equation editor with Word. Has to go through loops to finally enter an equation. One equation. That is it. After 10 minutes of using it, I uninstalled the whole thing, and also uninstalled indesign (was trial version). With no direct Latex support inside indesign, I mean direct support, indesign will never make it in science, math and engineering. Ok for magazines and articles. Nothing more.

    – Nasser
    Mar 4 '14 at 7:10














  • 7





    Without making any comparisons, there is Adobe inDesign

    – Scott H.
    Jun 20 '13 at 19:20






  • 10





    @ScottH. -- to the best of my knowledge, indesign doesn't (yet?) handle math.

    – barbara beeton
    Jun 20 '13 at 19:26






  • 3





    @barbarabeeton: But there is a plugin named MathMagic for Adobe InDesign to handle math.

    – kiss my armpit
    Jun 20 '13 at 19:55






  • 6





    There is an interesting comparison LaTeX vs. InDesign (but a bit old): zinktypografie.nl/latex.php?lang=en

    – student
    Jun 20 '13 at 20:06






  • 4





    I tried MathMagic plugin with inDesign. It was terrible. It is just like using the equation editor with Word. Has to go through loops to finally enter an equation. One equation. That is it. After 10 minutes of using it, I uninstalled the whole thing, and also uninstalled indesign (was trial version). With no direct Latex support inside indesign, I mean direct support, indesign will never make it in science, math and engineering. Ok for magazines and articles. Nothing more.

    – Nasser
    Mar 4 '14 at 7:10








7




7





Without making any comparisons, there is Adobe inDesign

– Scott H.
Jun 20 '13 at 19:20





Without making any comparisons, there is Adobe inDesign

– Scott H.
Jun 20 '13 at 19:20




10




10





@ScottH. -- to the best of my knowledge, indesign doesn't (yet?) handle math.

– barbara beeton
Jun 20 '13 at 19:26





@ScottH. -- to the best of my knowledge, indesign doesn't (yet?) handle math.

– barbara beeton
Jun 20 '13 at 19:26




3




3





@barbarabeeton: But there is a plugin named MathMagic for Adobe InDesign to handle math.

– kiss my armpit
Jun 20 '13 at 19:55





@barbarabeeton: But there is a plugin named MathMagic for Adobe InDesign to handle math.

– kiss my armpit
Jun 20 '13 at 19:55




6




6





There is an interesting comparison LaTeX vs. InDesign (but a bit old): zinktypografie.nl/latex.php?lang=en

– student
Jun 20 '13 at 20:06





There is an interesting comparison LaTeX vs. InDesign (but a bit old): zinktypografie.nl/latex.php?lang=en

– student
Jun 20 '13 at 20:06




4




4





I tried MathMagic plugin with inDesign. It was terrible. It is just like using the equation editor with Word. Has to go through loops to finally enter an equation. One equation. That is it. After 10 minutes of using it, I uninstalled the whole thing, and also uninstalled indesign (was trial version). With no direct Latex support inside indesign, I mean direct support, indesign will never make it in science, math and engineering. Ok for magazines and articles. Nothing more.

– Nasser
Mar 4 '14 at 7:10





I tried MathMagic plugin with inDesign. It was terrible. It is just like using the equation editor with Word. Has to go through loops to finally enter an equation. One equation. That is it. After 10 minutes of using it, I uninstalled the whole thing, and also uninstalled indesign (was trial version). With no direct Latex support inside indesign, I mean direct support, indesign will never make it in science, math and engineering. Ok for magazines and articles. Nothing more.

– Nasser
Mar 4 '14 at 7:10










13 Answers
13






active

oldest

votes


















55














You asked for professional solutions. Several top technical journals, including I believe Physical Review Letters and Nature magazine, use Advanced Print Publisher (APP), also known as Advent 3B2. Formerly available from Arbortext but now from PTC.



It can definitely handle mathematics, and I'm sure it deals with typographic details such as ligatures and microtype-like tweaks. I believe the main strength over tex is that it makes complete document production very automated. Things like dealing with floats (tables and figures) in a more-automatic way than tex, and giving manual controls to override it.



I haven't used it, however. (standard desktop version starts at US$5000-10000 as of January 2015). The wikipedia page has a little information.






share|improve this answer





















  • 47





    45,000 dollars? I wonder if they have student version?

    – Nasser
    Jun 22 '13 at 9:33






  • 3





    Does Advanced Print Publisher relate to LaTeX? I ask because I know submissions to Phys. Rev. Letters can be given in LaTeX (with the revtex package).

    – Chris Chudzicki
    Aug 4 '14 at 23:23






  • 3





    @mrc More likely if you submit in LaTeX, they send it to some developing country to re-type everything from scratch.

    – yo'
    Jan 14 '15 at 19:39






  • 1





    @yo' that would introduce many typos (in any country). I have seen myself submitting in TeX and getting edited documents (with punctuation and grammar corrections from the editor) very much like MSWord tracked changes that would be (or was) very difficult to do with LaTeX. I guess they have a battery of (human monitored) conversion tools. LaTeX <-> Word <-> Expensive proprietary system <-> LaTeX. The final quality is always pretty good in any case and it is hard to believe it is not LaTeX. I am talking about Physical Review here. There are other journals that obviously use only or mostly LaTeX

    – alfC
    Dec 17 '15 at 21:20



















142














I truly believe nothing comes close to TeX (and Friends, e.g. LaTeX, ConTeXt) when it comes to output quality.



Other DTP programs such as Indesign (Adobe), QuarkXPress (Quark) or Scribus (open source) offer a graphical interface and hence an easier learning curve, but they don't match up in terms of quality. Amongst many, let me just point the two most obvious quality advantages of TeX:



Math typesetting



Typesetting maths correctly is very complex (symbols need to adjust in size, spaces are very specific, etc.), and only TeX gets it right. There are probably even better examples, but try obtaining something like the following in InDesign:



math typesetting



Line and page breaking



Tex's algorithm is very complex and takes into account possible hyphens, widows, orphans, etc. to produce an output that is more elegant.



The best showcase of this I have found is a comparison posted by Roel Zinkstok of Zink Typography, reproduced below. On the comparison, Roel indicated with red dots lines with inter-word spacing that is out of proportion, a cardinal sin for typography, while the red circles indicate hyphens, which should be minimized:



line breaking






share|improve this answer





















  • 4





    @mozartstraße More precisely, the red dots represents the lines with inter-word spacing (IWS) larger than 9pt. The count is reflected in the table below the showcase.

    – Xavier
    Jun 20 '13 at 21:36






  • 11





    It also looks as though the pdfTeX output was made without the help of the microtype package.

    – jon
    Jun 20 '13 at 22:48






  • 15





    @ach You are probably thinking of the comments to this answer. The proof (as I would claim) that it's not a fair comparison can be found here, which shows the result with tex's default settings on the first page and then increasing tweaks to mimic the example. @jon Actually, I do not think this output is achieved by using microtype but rather by allowing spaces to stretch and shrink much more than tex would normally permit (being as fussy as it is on this point).

    – Robert
    Jun 20 '13 at 23:12








  • 2





    AFAIK, InDesign also have several very complex linebreaking algorithms. One should choose a proper algorithm and tweak the arguments to get proper output.

    – Leo Liu
    Jun 21 '13 at 4:27








  • 11





    @student: Adobe has stated openly (and in typeset form ;-) ) that InDesign uses precisely Knuth's linebreaking algorithms (per paragraph as opp. to per line). I have not used InDesign myself seriously, but from what I've seen done with it, I think it is a great typesetting system (ranking high together with QuarkXpress). Moreover, in this site there has been some discussion on grid-oriented typesetting design; maybe in this regard InDesign even ranks higher than TeX-based systems (not for lack of design excellence in TeX, but simply because it was not conceived with such a scenario in mind).

    – Marcos
    Jun 26 '13 at 18:06





















43














A shameless plug: We are offering a professional typesetter (though open source) which is based on LuaTeX. If it is "equal or even better" than LaTeX is surely subjective.



Our focus is to bring the DTP world to automatic typesetting (database publishing). For example we have




  • master pages based on arbitrary conditions

  • text flow based on "frames"

  • multi page tables including subtotals and repeating tablehead and tablefoot

  • easy to install/use fonts

  • grid based layout/typesetting

  • ...


While TeX / LaTeX is focused on text documents (with or without math) our main application area is product catalogs, price lists and other documents that are created automatically from databases but have flexible and nice layouts.



http://speedata.github.io/publisher/



I'd like to invite everyone to try out or perhaps work on the software. Development never ends (as usual).






share|improve this answer


























  • I'm intrigued by the grid-based support and text flowing. These are both regarded as 'hard' problems with TeX-based systems. How general are your solutions (in broad terms: not looking for anything commercially-sensitive)?

    – Joseph Wright
    Jul 1 '13 at 16:04











  • @JosephWright hard to discuss this in a comment... I believe your question is along the lines: "can we take the approach and throw it at LaTeX"? I'd say no, since I almost completely leave the TeX world. For example I save the text blocks and reformat them on demand (for example if columns are of different widths or of I need a parshape in the second column).

    – topskip
    Jul 1 '13 at 16:10






  • 16





    I wish I could get feedback for the downvote! Too much advertisement? (Hey, it's open source, so you can use it). Not a good answer? (I think it matches the question.) It would be nice if the downvoter could explain why this answer is a bad answer.

    – topskip
    Jul 2 '13 at 11:13













  • On 'generality' I was thinking for example that there are a few approaches to grid typesetting in TeX, but most (all?) have limitations, often related to display or math mode material. So I was wondering how generally you've been able to deal with this.

    – Joseph Wright
    Jul 2 '13 at 11:36











  • @JosephWright I have an invisible grid in the background with height of the grid cells of baselineskip of the text font. Every object is placed in the grid (and thus integer multiple of baselineskip from the top). There is still much to do but I will for example include an invisible "adjust to grid" marker that goes to the next cell. This part is currently under development.

    – topskip
    Jul 2 '13 at 11:46





















34














Troff (http://troff.org/) should definitely be mentioned for its simplicity. I normally use LaTeX for all my documents, but for some special cases (e.g. user manuals) troff (or it's GNU version groff) is just the simpler way. It also can handle math using the preprocessing tool eqn. A nice comparison between troff and LaTeX can be found here.






share|improve this answer



















  • 1





    I just realised that the link I provided for the comparison actually is a macro package for groff which the author claims makes it comparable to LaTeX. I think I just found something to do for the weekend :)

    – user2426172
    Jun 21 '13 at 8:14






  • 1





    Nice find! Can be a nice alternative to LaTeX for good looking automatically generated documents.

    – gniourf_gniourf
    Jun 21 '13 at 8:26






  • 1





    fyi, from wiki, troff can trace its origins back to a text formatting program called RUNOFF . see RUNOFF en.wikipedia.org/wiki/RUNOFF RUNOFF was the first computer text formatting program to see significant use.

    – Nasser
    Jun 22 '13 at 21:42











  • I used ditroff to typeset my thesis. And I believe groff has taken over some of $TeX$ line-handling and math formatting techniques, so it should be comparable for text at least. But overall a troff document feels like assembler to $LaTeX$' more structured approach. Lerned $'LaTeX$, and never looked back.

    – vonbrand
    Mar 23 '14 at 17:57








  • 1





    +1 for recommending Troff. For sophisticated typesetting, Heirloom Troff is a much better choice than Groff, offering support for microtypography, proper support for OpenType features, improved hyphenation, and - most importantly, native support for Type 1 fonts, TrueType and OpenType. Definitely a formidable contender with TeX for complex, accurate typesetting.

    – Alhadis
    Aug 12 '17 at 17:45



















33














GNU TeXMacs is a WYSIWYG scientific structured document editor and typesetter. It used to require a TeX distribution but now it does not.



Here is a list of not so widely known typesetting systems that I found googling around, mostly work-in-progress (or work-used-to-be-in-progress):




  • cl-typesetting is written in Common Lisp, which implies that its markup language is also Common Lisp. (Macro language, anyone?)


  • The Z format looks like HTML, but is not.


  • Platypus commands look like [list|bullet:{rarrow}]
    . Written in Java.



  • Patoline, written in OCaml. Also handles SQL bibliography databases, and comes with Bibi, a bibfile to SQL converter written in Haskell. (The developers seem to be really keen on functional languages.) You have to build it from source. Commands look like a mixture of TeX, OCaml and new markup:



    (* #FORMAT FormatArticle *)
    begin{genumerate}(AlphaLower, fun s -> [tT (s^". ")])
    item First item
    item Second item
    end{genumerate}
    includeGraphics("pato.png")
    $$ a + b $$
    $ a + b $
    Caml(
    let dr ()=
    [bB (fun _->
    [Drawing (drawing [Path (default,[rectangle (0.,0.) (10.,10.)])])]
    )]
    )


    Note that I have no idea if this code works, I just copied together parts of the manual.




  • SILE (Simon's Improved Layout Engine) is written in Lua with the Pango and Cairo libraries. It also tries to provide a syntax nearly identical to that of LaTeX:



    begin[class=book,papersize=129mm x 198mm]{document}
    include[src=examples/macros.sil]
    script[src=packages/grid]
    left-running-head{A Scandal in Bohemia}
    right-running-head{hfill{}Alexander Conan Doyle}
    headline{A Scandal In Bohemia}

    noindent To Sherlock Holmes she is always em{the woman}.
    footnote{Lorem ipsum dolor sit amet.}

    begin[family=Snell Roundhand]{font}
    Consequetur adipiscing elit.
    end{font}
    end{document}


  • Rinohtype, written in Python. The input format is reStructuredText. See an example input file.



Maybe more to come.






share|improve this answer





















  • 1





    AFAIK, TeXmacs was never based on TeX; it uses various TeX algorithms but they are implemented independent of TeX. It uses TeX fonts, though, so that might be why it needed a TeX distribution in the past.

    – Khaled Hosny
    Jul 1 '13 at 22:49













  • Nice edit, I searched for this just to add it :) But it seems that you already added it.

    – Manuel
    Aug 3 '14 at 21:28






  • 2





    This Patoline thing looks promising, in case of active development. It can provide better error messages than TeX/LaTeX, due to types and safety checking.

    – Anton Kochkov
    Aug 19 '14 at 13:16











  • The Patoline manual has not been available for about half a year (patoline.org/patobook.pdf), apparently because of a bug in Patoline, sadly.

    – Benjamin McKay
    Jan 11 '15 at 16:26



















20














First, add XyVision to the list of proprietary batch composition competitors.



Here's a bit of commentary I wrote up a couple of years ago, which still applies. It's also on the ConTeXt wiki.



https://groups.google.com/forum/message/raw?msg=comp.text.tex/8zugdUxw6dI/mu7Qzs4bQDYJ



So,




  • using Quark is like being chained to an oar which is covered w/
    splinters and mostly broken at the other end and which will randomly
    break due to being poorly carved (Quark has crashed on me 183 times
    this year) leaving one adrift or run aground, or sometimes returning
    the vessel to its starting point (a few of those crashes have resulted
    in unrecoverable document corruption --- my autobackup folder may
    contain 2 or 3 GBs of files for a given iteration of a particular
    project each month) --- the oar can be smoothed somewhat and reinforced
    (by purchasing or finding XTensions, using XTags &c.) and periodically
    one is required to purchase a new oar (sometimes just after the
    previous one has been customized adequately). For some tasks, one can
    impress any graphic designer as a galley slave to ease the effort for
    others, but while charts are available, there are not automagic
    navigation options and every journey must be manually piloted.


  • using InDesign is pretty much the same except the oar is smoother
    and stronger (it's crashed 29 times on me thus far this year), there
    aren't as many customization options and it's not quite as easy to find
    a candidate for impressment (though soon it'll be as easy as for
    Quark). Charts are available, but again, piloting is strictly manual.


  • using Plain TeX one has to craft the vessel's oar oneself (as well
    as the rest of the vessel unless one is typesetting a clone of The
    TeXbook
    ), but it's as sturdy and as nice a one as one's skills allow
    and can even be an engine which moves the vessel in and of itself ---
    it can be difficult or impossible to find people suitable to help w/
    either carving the oar or using it though, but once a given journey is
    worked out, the oar becomes magical and rows for itself except for when
    one runs into an unplanned for obstacle (the navigation charts are old
    ones and not often up-dated, with a lot of ``terra incognita''),
    allowing one an auto-pilot option for certain journeys, dependent upon
    one's skill.


  • using ePlain, an oar is provided, can be customized, and can be
    enchanted and the charts are okay, but have a lot of ``terra
    incognita'' on them.


  • using LaTeX, an oar is provided and there're lots of nifty
    customizations and improvements already available, and one can impress
    additional oars from CTAN, however on a semi-random basis, adding one
    oar will break other oars, sometimes leaving one adrift or run aground.
    One can enchant a set of oars to accomplish a given journey, easing the
    piloting requirement, and the navigation charts are decent and
    obstacles are fairly well-known.


  • using ConTeXt, a very nice oar is provided, which has lots of
    customization options, but the navigational charts aren't easily read
    by a traditionally trained navigator at first, although they are fairly
    compleat and most journey can be carefully worked out, but once one is,
    it is quite automatic and there's a good auto-pilot option.







share|improve this answer



















  • 7





    your quotes around terra incognita give away your preference :P

    – percusse
    Jun 28 '13 at 23:33






  • 1





    +1 for style: certainly puts LaTex in context better than the usual table-of-features-I-then-need-to-look-up.

    – WillC
    Aug 29 '18 at 6:32



















19














A new alternative is Patoline, which is still in an early development stage but the goal is to be a modern digital typesetting system.



As for the LaTeX compatibility one can describe the Patoline compiler as




Obviously, a related (but much smaller) project is the Patoline compiler, which compiles a mixed Wiki/LaTeX/Ocaml syntax into a variety of output formats, including traditional PDF files, but also web servers that deliver dynamic contents, synchronized for instance with a talk.




The documentation includes a PhD thesis and a description of the syntax which was produced using Patoline, and which looks to me as an amateur in typesetting pretty good:



enter image description here






share|improve this answer


























  • However, Patoline has many flaws that make it very hard to use at the moment. As well, it introduces some concepts that I don't quite like. I wonder if Xavier and his collaborators manage to make it as tweakable as TeX is, in the sense that any feature it has has to be tunable, and in particular, easy to disable.

    – yo'
    Aug 19 '14 at 14:03






  • 1





    @tohecz: I have unfortunately not found the time to try it out myself. Still, I am hoping for an alternative to LaTeX which keeps the good features but replaces the 'programming language' with something more reasonable. LuaTeX is definitely a step in the right direction.

    – Alexander
    Aug 20 '14 at 13:37






  • 2





    That's another point: OCaml doesn't look like a good choice...

    – yo'
    Aug 20 '14 at 14:08













  • sorry I used the wrong name in the 1st comment, the main developer is of course Pierre-Etienne, not Xavier.

    – yo'
    Aug 20 '14 at 14:13



















16














While LaTeX is the markup language of choice in many scientific and scholarly circles (especially in the fields of mathematics, computer science, physics, and astronomy), some scholars are starting to write papers using new markup languages tailored for the web. Markdown is one of them. It is very versatile and it can be easily extended to satisfy the needs of scientific writing. The advantage of using Markdown (or similar) instead of LaTeX is that we are writing on the web more than ever. Thus, it makes sense to use a web native typesetting format. For those who would like to use both Markdown and Latex on the web, Authorea (full disclosure, I am a co-founder) is an online social authoring platform that allows articles to be edited in both formats simultaneously. Authorea also renders and compiles LaTeX and MathML equations to the web, in addition to PDF.






share|improve this answer





















  • 3





    LaTeX is not only markup but also typesetting. And welcome to TeX.sx!

    – Qrrbrbirlbel
    Jun 25 '13 at 22:56











  • Welcome to TeX.sx! That platform looks quite intriguing, indeed. Do you have any relationship to the company that runs it (apart from being a user)?

    – Jake
    Jun 26 '13 at 1:31











  • hi Jake- yes I am the co-founder of Authorea (and a user, since I still write papers in Astronomy + Information Science). We have a couple of thousand users and the vast majority of them writes in LaTeX.

    – Alberto Pepe
    Jun 26 '13 at 15:44











  • @user32799: Thanks for the reply. When posting an answer promoting a product you're affiliated with, that affiliation should be disclosed (see tex.stackexchange.com/help/behavior). Could you edit your post to include that information?

    – Jake
    Jun 28 '13 at 20:50











  • hi Jake- I just updated my .sx profile information with my affiliation. Thanks!

    – Alberto Pepe
    Jun 29 '13 at 21:02



















14














TeX and friends maybe still the best. There are some alternative and results are various. I do not really use any of them, but kind of know there existence.




  • Lout

  • ANT






share|improve this answer



















  • 2





    Another one: asciidoc

    – qunying
    Jun 20 '13 at 23:34








  • 2





    there was also chi-writer. no longer around. en.wikipedia.org/wiki/ChiWriter

    – Nasser
    Jun 20 '13 at 23:44






  • 3





    ANT is an impressive codebase, especially the support they have for OTF. Also, the embedded scripting languages is a huge win. Unfortunately it depends on archaic versions of libraries and the language, so it won’t build on current systems. The most recent post on their mailing list is from 2010 so I guess it could be considered unmaintained.

    – Philipp Gesang
    Jun 21 '13 at 9:10






  • 3





    I'm really interested in whether Lout and Ant have ever been used. Can you give examples/sample output if there's any? (Apart from their respective user manuals, of course.) I'm also interested in their "stories", why they are unmaintained, what happened with the author, etc.

    – marczellm
    Jun 22 '13 at 8:37











  • Follow the example in Lout's user manual will do. Lout is still under development, it just released 3.40. Jeffrey (the author) does not use source version control. But you could always look at the mailing list, it is a low volume list but active. He plan his next typesetting engine Nonpareil, but seems not much news yet. Main drawback that Lout doesn't support unicode. Not sure about ANT.

    – qunying
    Jun 26 '13 at 17:23



















10














As long as you don't need mathematical typesetting, you actually can find better than TeX with Heirloom Documentation Tools. Not only does it provide Knuth's algorithm for formatting paragraphs; it also allows to compute spacing by mixing three systems (interletters spacing, interwords spacing, imperceptible change in the shapes of the glyphs). Thus you can work with constant space between words! Selecting a font among various kinds of fonts is much easier and quicker than with TeX. I have been using it for years while my colleagues use LaTeX; they wouldn't have the idea to only think their documents can be on a par with mine. If you are interested in typography, then Heirloom version of troff is definitely the way to do.



edit: Since I first wrote this answer, it looks like new pieces of software have appeared. I just heard about utroff which seems to come from Heirloom troff; I will very certainly have a look at it and probably use it now because Heirloom troff doesn't seem to be actively maintained and may be difficult to compile.






share|improve this answer

































    1














    Scientific Word can be another alternative software for scientists:
    http://www.mackichan.com/index.html?products/sw.html~mainFrame






    share|improve this answer





















    • 3





      Scientific Word comprises a TeX distribution along with a quasi-WYSIWYG front end. The code it generates is (mostly) LaTeX. I'm thus doubtful that Scientific Word can be considered a alternative to LaTeX. Separately, the software hasn't seen an upgrade in years and years. The current version, numbered 5.5, has been around since at least 2007 (and probably even longer). A software package that's been around essentially unchanged for more than seven years is functionally obsolete, I'm afraid.

      – Mico
      Jan 11 '15 at 17:22











    • @Mico As far as I agree, the same thing can essentially be said about TeX.

      – Gaussler
      Jul 11 '15 at 14:27











    • @Gaussler - Version 6 of Scientific Word has been released quite recently; I'm sure the long-suffering users of SW were jubilant. I don't agree that SW's situation is comparable to that of TeX. (i) TeXLive has been every year for many years now. (ii) While "Plain TeX" is frozen, that's not the case for pdfLaTeX, XeTeX, LuaTeX, and lots and lots of packages for pdfLaTeX, XeLaTeX, and LuaTeX. (iii) LaTeX2e was also nearly frozen for many years, but got a significant update as of 1/1/2015.

      – Mico
      Jul 11 '15 at 14:54













    • @Mico Still, the pdfTeX backend has remained dominant and frozen since forever. Also, you compare Plain TeX - a format - to LuaTeX and XeTeX - two engines.

      – Gaussler
      Jul 11 '15 at 15:01








    • 1





      @Gaussler - I'm no good at linguistic quibbles. Can't vouch for what "standard" terminology may be. All I can hope for is that I've clarified what I meant to say.

      – Mico
      Jul 11 '15 at 15:25





















    1














    Pollen, written in Racket (of the Lisp family). The primary focus is web-based typography (and no math so far), but it can also support other formats such as pdf.






    share|improve this answer































      0














      R Markdown runs on the TeX distribution when using kntr to create PDFs. But, to type in it requires less code memorization since R produces default YMAL headers when you load up a new document.






      share|improve this answer








      New contributor




      Jennifer is a new contributor to this site. Take care in asking for clarification, commenting, and answering.
      Check out our Code of Conduct.



















        protected by yo' Aug 19 '14 at 14:03



        Thank you for your interest in this question.
        Because it has attracted low-quality or spam answers that had to be removed, posting an answer now requires 10 reputation on this site (the association bonus does not count).



        Would you like to answer one of these unanswered questions instead?














        13 Answers
        13






        active

        oldest

        votes








        13 Answers
        13






        active

        oldest

        votes









        active

        oldest

        votes






        active

        oldest

        votes









        55














        You asked for professional solutions. Several top technical journals, including I believe Physical Review Letters and Nature magazine, use Advanced Print Publisher (APP), also known as Advent 3B2. Formerly available from Arbortext but now from PTC.



        It can definitely handle mathematics, and I'm sure it deals with typographic details such as ligatures and microtype-like tweaks. I believe the main strength over tex is that it makes complete document production very automated. Things like dealing with floats (tables and figures) in a more-automatic way than tex, and giving manual controls to override it.



        I haven't used it, however. (standard desktop version starts at US$5000-10000 as of January 2015). The wikipedia page has a little information.






        share|improve this answer





















        • 47





          45,000 dollars? I wonder if they have student version?

          – Nasser
          Jun 22 '13 at 9:33






        • 3





          Does Advanced Print Publisher relate to LaTeX? I ask because I know submissions to Phys. Rev. Letters can be given in LaTeX (with the revtex package).

          – Chris Chudzicki
          Aug 4 '14 at 23:23






        • 3





          @mrc More likely if you submit in LaTeX, they send it to some developing country to re-type everything from scratch.

          – yo'
          Jan 14 '15 at 19:39






        • 1





          @yo' that would introduce many typos (in any country). I have seen myself submitting in TeX and getting edited documents (with punctuation and grammar corrections from the editor) very much like MSWord tracked changes that would be (or was) very difficult to do with LaTeX. I guess they have a battery of (human monitored) conversion tools. LaTeX <-> Word <-> Expensive proprietary system <-> LaTeX. The final quality is always pretty good in any case and it is hard to believe it is not LaTeX. I am talking about Physical Review here. There are other journals that obviously use only or mostly LaTeX

          – alfC
          Dec 17 '15 at 21:20
















        55














        You asked for professional solutions. Several top technical journals, including I believe Physical Review Letters and Nature magazine, use Advanced Print Publisher (APP), also known as Advent 3B2. Formerly available from Arbortext but now from PTC.



        It can definitely handle mathematics, and I'm sure it deals with typographic details such as ligatures and microtype-like tweaks. I believe the main strength over tex is that it makes complete document production very automated. Things like dealing with floats (tables and figures) in a more-automatic way than tex, and giving manual controls to override it.



        I haven't used it, however. (standard desktop version starts at US$5000-10000 as of January 2015). The wikipedia page has a little information.






        share|improve this answer





















        • 47





          45,000 dollars? I wonder if they have student version?

          – Nasser
          Jun 22 '13 at 9:33






        • 3





          Does Advanced Print Publisher relate to LaTeX? I ask because I know submissions to Phys. Rev. Letters can be given in LaTeX (with the revtex package).

          – Chris Chudzicki
          Aug 4 '14 at 23:23






        • 3





          @mrc More likely if you submit in LaTeX, they send it to some developing country to re-type everything from scratch.

          – yo'
          Jan 14 '15 at 19:39






        • 1





          @yo' that would introduce many typos (in any country). I have seen myself submitting in TeX and getting edited documents (with punctuation and grammar corrections from the editor) very much like MSWord tracked changes that would be (or was) very difficult to do with LaTeX. I guess they have a battery of (human monitored) conversion tools. LaTeX <-> Word <-> Expensive proprietary system <-> LaTeX. The final quality is always pretty good in any case and it is hard to believe it is not LaTeX. I am talking about Physical Review here. There are other journals that obviously use only or mostly LaTeX

          – alfC
          Dec 17 '15 at 21:20














        55












        55








        55







        You asked for professional solutions. Several top technical journals, including I believe Physical Review Letters and Nature magazine, use Advanced Print Publisher (APP), also known as Advent 3B2. Formerly available from Arbortext but now from PTC.



        It can definitely handle mathematics, and I'm sure it deals with typographic details such as ligatures and microtype-like tweaks. I believe the main strength over tex is that it makes complete document production very automated. Things like dealing with floats (tables and figures) in a more-automatic way than tex, and giving manual controls to override it.



        I haven't used it, however. (standard desktop version starts at US$5000-10000 as of January 2015). The wikipedia page has a little information.






        share|improve this answer















        You asked for professional solutions. Several top technical journals, including I believe Physical Review Letters and Nature magazine, use Advanced Print Publisher (APP), also known as Advent 3B2. Formerly available from Arbortext but now from PTC.



        It can definitely handle mathematics, and I'm sure it deals with typographic details such as ligatures and microtype-like tweaks. I believe the main strength over tex is that it makes complete document production very automated. Things like dealing with floats (tables and figures) in a more-automatic way than tex, and giving manual controls to override it.



        I haven't used it, however. (standard desktop version starts at US$5000-10000 as of January 2015). The wikipedia page has a little information.







        share|improve this answer














        share|improve this answer



        share|improve this answer








        edited Jan 14 '15 at 19:38









        yo'

        39.8k8124233




        39.8k8124233










        answered Jun 22 '13 at 3:46









        Lev BishopLev Bishop

        37.2k794150




        37.2k794150








        • 47





          45,000 dollars? I wonder if they have student version?

          – Nasser
          Jun 22 '13 at 9:33






        • 3





          Does Advanced Print Publisher relate to LaTeX? I ask because I know submissions to Phys. Rev. Letters can be given in LaTeX (with the revtex package).

          – Chris Chudzicki
          Aug 4 '14 at 23:23






        • 3





          @mrc More likely if you submit in LaTeX, they send it to some developing country to re-type everything from scratch.

          – yo'
          Jan 14 '15 at 19:39






        • 1





          @yo' that would introduce many typos (in any country). I have seen myself submitting in TeX and getting edited documents (with punctuation and grammar corrections from the editor) very much like MSWord tracked changes that would be (or was) very difficult to do with LaTeX. I guess they have a battery of (human monitored) conversion tools. LaTeX <-> Word <-> Expensive proprietary system <-> LaTeX. The final quality is always pretty good in any case and it is hard to believe it is not LaTeX. I am talking about Physical Review here. There are other journals that obviously use only or mostly LaTeX

          – alfC
          Dec 17 '15 at 21:20














        • 47





          45,000 dollars? I wonder if they have student version?

          – Nasser
          Jun 22 '13 at 9:33






        • 3





          Does Advanced Print Publisher relate to LaTeX? I ask because I know submissions to Phys. Rev. Letters can be given in LaTeX (with the revtex package).

          – Chris Chudzicki
          Aug 4 '14 at 23:23






        • 3





          @mrc More likely if you submit in LaTeX, they send it to some developing country to re-type everything from scratch.

          – yo'
          Jan 14 '15 at 19:39






        • 1





          @yo' that would introduce many typos (in any country). I have seen myself submitting in TeX and getting edited documents (with punctuation and grammar corrections from the editor) very much like MSWord tracked changes that would be (or was) very difficult to do with LaTeX. I guess they have a battery of (human monitored) conversion tools. LaTeX <-> Word <-> Expensive proprietary system <-> LaTeX. The final quality is always pretty good in any case and it is hard to believe it is not LaTeX. I am talking about Physical Review here. There are other journals that obviously use only or mostly LaTeX

          – alfC
          Dec 17 '15 at 21:20








        47




        47





        45,000 dollars? I wonder if they have student version?

        – Nasser
        Jun 22 '13 at 9:33





        45,000 dollars? I wonder if they have student version?

        – Nasser
        Jun 22 '13 at 9:33




        3




        3





        Does Advanced Print Publisher relate to LaTeX? I ask because I know submissions to Phys. Rev. Letters can be given in LaTeX (with the revtex package).

        – Chris Chudzicki
        Aug 4 '14 at 23:23





        Does Advanced Print Publisher relate to LaTeX? I ask because I know submissions to Phys. Rev. Letters can be given in LaTeX (with the revtex package).

        – Chris Chudzicki
        Aug 4 '14 at 23:23




        3




        3





        @mrc More likely if you submit in LaTeX, they send it to some developing country to re-type everything from scratch.

        – yo'
        Jan 14 '15 at 19:39





        @mrc More likely if you submit in LaTeX, they send it to some developing country to re-type everything from scratch.

        – yo'
        Jan 14 '15 at 19:39




        1




        1





        @yo' that would introduce many typos (in any country). I have seen myself submitting in TeX and getting edited documents (with punctuation and grammar corrections from the editor) very much like MSWord tracked changes that would be (or was) very difficult to do with LaTeX. I guess they have a battery of (human monitored) conversion tools. LaTeX <-> Word <-> Expensive proprietary system <-> LaTeX. The final quality is always pretty good in any case and it is hard to believe it is not LaTeX. I am talking about Physical Review here. There are other journals that obviously use only or mostly LaTeX

        – alfC
        Dec 17 '15 at 21:20





        @yo' that would introduce many typos (in any country). I have seen myself submitting in TeX and getting edited documents (with punctuation and grammar corrections from the editor) very much like MSWord tracked changes that would be (or was) very difficult to do with LaTeX. I guess they have a battery of (human monitored) conversion tools. LaTeX <-> Word <-> Expensive proprietary system <-> LaTeX. The final quality is always pretty good in any case and it is hard to believe it is not LaTeX. I am talking about Physical Review here. There are other journals that obviously use only or mostly LaTeX

        – alfC
        Dec 17 '15 at 21:20











        142














        I truly believe nothing comes close to TeX (and Friends, e.g. LaTeX, ConTeXt) when it comes to output quality.



        Other DTP programs such as Indesign (Adobe), QuarkXPress (Quark) or Scribus (open source) offer a graphical interface and hence an easier learning curve, but they don't match up in terms of quality. Amongst many, let me just point the two most obvious quality advantages of TeX:



        Math typesetting



        Typesetting maths correctly is very complex (symbols need to adjust in size, spaces are very specific, etc.), and only TeX gets it right. There are probably even better examples, but try obtaining something like the following in InDesign:



        math typesetting



        Line and page breaking



        Tex's algorithm is very complex and takes into account possible hyphens, widows, orphans, etc. to produce an output that is more elegant.



        The best showcase of this I have found is a comparison posted by Roel Zinkstok of Zink Typography, reproduced below. On the comparison, Roel indicated with red dots lines with inter-word spacing that is out of proportion, a cardinal sin for typography, while the red circles indicate hyphens, which should be minimized:



        line breaking






        share|improve this answer





















        • 4





          @mozartstraße More precisely, the red dots represents the lines with inter-word spacing (IWS) larger than 9pt. The count is reflected in the table below the showcase.

          – Xavier
          Jun 20 '13 at 21:36






        • 11





          It also looks as though the pdfTeX output was made without the help of the microtype package.

          – jon
          Jun 20 '13 at 22:48






        • 15





          @ach You are probably thinking of the comments to this answer. The proof (as I would claim) that it's not a fair comparison can be found here, which shows the result with tex's default settings on the first page and then increasing tweaks to mimic the example. @jon Actually, I do not think this output is achieved by using microtype but rather by allowing spaces to stretch and shrink much more than tex would normally permit (being as fussy as it is on this point).

          – Robert
          Jun 20 '13 at 23:12








        • 2





          AFAIK, InDesign also have several very complex linebreaking algorithms. One should choose a proper algorithm and tweak the arguments to get proper output.

          – Leo Liu
          Jun 21 '13 at 4:27








        • 11





          @student: Adobe has stated openly (and in typeset form ;-) ) that InDesign uses precisely Knuth's linebreaking algorithms (per paragraph as opp. to per line). I have not used InDesign myself seriously, but from what I've seen done with it, I think it is a great typesetting system (ranking high together with QuarkXpress). Moreover, in this site there has been some discussion on grid-oriented typesetting design; maybe in this regard InDesign even ranks higher than TeX-based systems (not for lack of design excellence in TeX, but simply because it was not conceived with such a scenario in mind).

          – Marcos
          Jun 26 '13 at 18:06


















        142














        I truly believe nothing comes close to TeX (and Friends, e.g. LaTeX, ConTeXt) when it comes to output quality.



        Other DTP programs such as Indesign (Adobe), QuarkXPress (Quark) or Scribus (open source) offer a graphical interface and hence an easier learning curve, but they don't match up in terms of quality. Amongst many, let me just point the two most obvious quality advantages of TeX:



        Math typesetting



        Typesetting maths correctly is very complex (symbols need to adjust in size, spaces are very specific, etc.), and only TeX gets it right. There are probably even better examples, but try obtaining something like the following in InDesign:



        math typesetting



        Line and page breaking



        Tex's algorithm is very complex and takes into account possible hyphens, widows, orphans, etc. to produce an output that is more elegant.



        The best showcase of this I have found is a comparison posted by Roel Zinkstok of Zink Typography, reproduced below. On the comparison, Roel indicated with red dots lines with inter-word spacing that is out of proportion, a cardinal sin for typography, while the red circles indicate hyphens, which should be minimized:



        line breaking






        share|improve this answer





















        • 4





          @mozartstraße More precisely, the red dots represents the lines with inter-word spacing (IWS) larger than 9pt. The count is reflected in the table below the showcase.

          – Xavier
          Jun 20 '13 at 21:36






        • 11





          It also looks as though the pdfTeX output was made without the help of the microtype package.

          – jon
          Jun 20 '13 at 22:48






        • 15





          @ach You are probably thinking of the comments to this answer. The proof (as I would claim) that it's not a fair comparison can be found here, which shows the result with tex's default settings on the first page and then increasing tweaks to mimic the example. @jon Actually, I do not think this output is achieved by using microtype but rather by allowing spaces to stretch and shrink much more than tex would normally permit (being as fussy as it is on this point).

          – Robert
          Jun 20 '13 at 23:12








        • 2





          AFAIK, InDesign also have several very complex linebreaking algorithms. One should choose a proper algorithm and tweak the arguments to get proper output.

          – Leo Liu
          Jun 21 '13 at 4:27








        • 11





          @student: Adobe has stated openly (and in typeset form ;-) ) that InDesign uses precisely Knuth's linebreaking algorithms (per paragraph as opp. to per line). I have not used InDesign myself seriously, but from what I've seen done with it, I think it is a great typesetting system (ranking high together with QuarkXpress). Moreover, in this site there has been some discussion on grid-oriented typesetting design; maybe in this regard InDesign even ranks higher than TeX-based systems (not for lack of design excellence in TeX, but simply because it was not conceived with such a scenario in mind).

          – Marcos
          Jun 26 '13 at 18:06
















        142












        142








        142







        I truly believe nothing comes close to TeX (and Friends, e.g. LaTeX, ConTeXt) when it comes to output quality.



        Other DTP programs such as Indesign (Adobe), QuarkXPress (Quark) or Scribus (open source) offer a graphical interface and hence an easier learning curve, but they don't match up in terms of quality. Amongst many, let me just point the two most obvious quality advantages of TeX:



        Math typesetting



        Typesetting maths correctly is very complex (symbols need to adjust in size, spaces are very specific, etc.), and only TeX gets it right. There are probably even better examples, but try obtaining something like the following in InDesign:



        math typesetting



        Line and page breaking



        Tex's algorithm is very complex and takes into account possible hyphens, widows, orphans, etc. to produce an output that is more elegant.



        The best showcase of this I have found is a comparison posted by Roel Zinkstok of Zink Typography, reproduced below. On the comparison, Roel indicated with red dots lines with inter-word spacing that is out of proportion, a cardinal sin for typography, while the red circles indicate hyphens, which should be minimized:



        line breaking






        share|improve this answer















        I truly believe nothing comes close to TeX (and Friends, e.g. LaTeX, ConTeXt) when it comes to output quality.



        Other DTP programs such as Indesign (Adobe), QuarkXPress (Quark) or Scribus (open source) offer a graphical interface and hence an easier learning curve, but they don't match up in terms of quality. Amongst many, let me just point the two most obvious quality advantages of TeX:



        Math typesetting



        Typesetting maths correctly is very complex (symbols need to adjust in size, spaces are very specific, etc.), and only TeX gets it right. There are probably even better examples, but try obtaining something like the following in InDesign:



        math typesetting



        Line and page breaking



        Tex's algorithm is very complex and takes into account possible hyphens, widows, orphans, etc. to produce an output that is more elegant.



        The best showcase of this I have found is a comparison posted by Roel Zinkstok of Zink Typography, reproduced below. On the comparison, Roel indicated with red dots lines with inter-word spacing that is out of proportion, a cardinal sin for typography, while the red circles indicate hyphens, which should be minimized:



        line breaking







        share|improve this answer














        share|improve this answer



        share|improve this answer








        edited Jun 28 '13 at 20:36

























        answered Jun 20 '13 at 20:20









        XavierXavier

        10.8k64772




        10.8k64772








        • 4





          @mozartstraße More precisely, the red dots represents the lines with inter-word spacing (IWS) larger than 9pt. The count is reflected in the table below the showcase.

          – Xavier
          Jun 20 '13 at 21:36






        • 11





          It also looks as though the pdfTeX output was made without the help of the microtype package.

          – jon
          Jun 20 '13 at 22:48






        • 15





          @ach You are probably thinking of the comments to this answer. The proof (as I would claim) that it's not a fair comparison can be found here, which shows the result with tex's default settings on the first page and then increasing tweaks to mimic the example. @jon Actually, I do not think this output is achieved by using microtype but rather by allowing spaces to stretch and shrink much more than tex would normally permit (being as fussy as it is on this point).

          – Robert
          Jun 20 '13 at 23:12








        • 2





          AFAIK, InDesign also have several very complex linebreaking algorithms. One should choose a proper algorithm and tweak the arguments to get proper output.

          – Leo Liu
          Jun 21 '13 at 4:27








        • 11





          @student: Adobe has stated openly (and in typeset form ;-) ) that InDesign uses precisely Knuth's linebreaking algorithms (per paragraph as opp. to per line). I have not used InDesign myself seriously, but from what I've seen done with it, I think it is a great typesetting system (ranking high together with QuarkXpress). Moreover, in this site there has been some discussion on grid-oriented typesetting design; maybe in this regard InDesign even ranks higher than TeX-based systems (not for lack of design excellence in TeX, but simply because it was not conceived with such a scenario in mind).

          – Marcos
          Jun 26 '13 at 18:06
















        • 4





          @mozartstraße More precisely, the red dots represents the lines with inter-word spacing (IWS) larger than 9pt. The count is reflected in the table below the showcase.

          – Xavier
          Jun 20 '13 at 21:36






        • 11





          It also looks as though the pdfTeX output was made without the help of the microtype package.

          – jon
          Jun 20 '13 at 22:48






        • 15





          @ach You are probably thinking of the comments to this answer. The proof (as I would claim) that it's not a fair comparison can be found here, which shows the result with tex's default settings on the first page and then increasing tweaks to mimic the example. @jon Actually, I do not think this output is achieved by using microtype but rather by allowing spaces to stretch and shrink much more than tex would normally permit (being as fussy as it is on this point).

          – Robert
          Jun 20 '13 at 23:12








        • 2





          AFAIK, InDesign also have several very complex linebreaking algorithms. One should choose a proper algorithm and tweak the arguments to get proper output.

          – Leo Liu
          Jun 21 '13 at 4:27








        • 11





          @student: Adobe has stated openly (and in typeset form ;-) ) that InDesign uses precisely Knuth's linebreaking algorithms (per paragraph as opp. to per line). I have not used InDesign myself seriously, but from what I've seen done with it, I think it is a great typesetting system (ranking high together with QuarkXpress). Moreover, in this site there has been some discussion on grid-oriented typesetting design; maybe in this regard InDesign even ranks higher than TeX-based systems (not for lack of design excellence in TeX, but simply because it was not conceived with such a scenario in mind).

          – Marcos
          Jun 26 '13 at 18:06










        4




        4





        @mozartstraße More precisely, the red dots represents the lines with inter-word spacing (IWS) larger than 9pt. The count is reflected in the table below the showcase.

        – Xavier
        Jun 20 '13 at 21:36





        @mozartstraße More precisely, the red dots represents the lines with inter-word spacing (IWS) larger than 9pt. The count is reflected in the table below the showcase.

        – Xavier
        Jun 20 '13 at 21:36




        11




        11





        It also looks as though the pdfTeX output was made without the help of the microtype package.

        – jon
        Jun 20 '13 at 22:48





        It also looks as though the pdfTeX output was made without the help of the microtype package.

        – jon
        Jun 20 '13 at 22:48




        15




        15





        @ach You are probably thinking of the comments to this answer. The proof (as I would claim) that it's not a fair comparison can be found here, which shows the result with tex's default settings on the first page and then increasing tweaks to mimic the example. @jon Actually, I do not think this output is achieved by using microtype but rather by allowing spaces to stretch and shrink much more than tex would normally permit (being as fussy as it is on this point).

        – Robert
        Jun 20 '13 at 23:12







        @ach You are probably thinking of the comments to this answer. The proof (as I would claim) that it's not a fair comparison can be found here, which shows the result with tex's default settings on the first page and then increasing tweaks to mimic the example. @jon Actually, I do not think this output is achieved by using microtype but rather by allowing spaces to stretch and shrink much more than tex would normally permit (being as fussy as it is on this point).

        – Robert
        Jun 20 '13 at 23:12






        2




        2





        AFAIK, InDesign also have several very complex linebreaking algorithms. One should choose a proper algorithm and tweak the arguments to get proper output.

        – Leo Liu
        Jun 21 '13 at 4:27







        AFAIK, InDesign also have several very complex linebreaking algorithms. One should choose a proper algorithm and tweak the arguments to get proper output.

        – Leo Liu
        Jun 21 '13 at 4:27






        11




        11





        @student: Adobe has stated openly (and in typeset form ;-) ) that InDesign uses precisely Knuth's linebreaking algorithms (per paragraph as opp. to per line). I have not used InDesign myself seriously, but from what I've seen done with it, I think it is a great typesetting system (ranking high together with QuarkXpress). Moreover, in this site there has been some discussion on grid-oriented typesetting design; maybe in this regard InDesign even ranks higher than TeX-based systems (not for lack of design excellence in TeX, but simply because it was not conceived with such a scenario in mind).

        – Marcos
        Jun 26 '13 at 18:06







        @student: Adobe has stated openly (and in typeset form ;-) ) that InDesign uses precisely Knuth's linebreaking algorithms (per paragraph as opp. to per line). I have not used InDesign myself seriously, but from what I've seen done with it, I think it is a great typesetting system (ranking high together with QuarkXpress). Moreover, in this site there has been some discussion on grid-oriented typesetting design; maybe in this regard InDesign even ranks higher than TeX-based systems (not for lack of design excellence in TeX, but simply because it was not conceived with such a scenario in mind).

        – Marcos
        Jun 26 '13 at 18:06













        43














        A shameless plug: We are offering a professional typesetter (though open source) which is based on LuaTeX. If it is "equal or even better" than LaTeX is surely subjective.



        Our focus is to bring the DTP world to automatic typesetting (database publishing). For example we have




        • master pages based on arbitrary conditions

        • text flow based on "frames"

        • multi page tables including subtotals and repeating tablehead and tablefoot

        • easy to install/use fonts

        • grid based layout/typesetting

        • ...


        While TeX / LaTeX is focused on text documents (with or without math) our main application area is product catalogs, price lists and other documents that are created automatically from databases but have flexible and nice layouts.



        http://speedata.github.io/publisher/



        I'd like to invite everyone to try out or perhaps work on the software. Development never ends (as usual).






        share|improve this answer


























        • I'm intrigued by the grid-based support and text flowing. These are both regarded as 'hard' problems with TeX-based systems. How general are your solutions (in broad terms: not looking for anything commercially-sensitive)?

          – Joseph Wright
          Jul 1 '13 at 16:04











        • @JosephWright hard to discuss this in a comment... I believe your question is along the lines: "can we take the approach and throw it at LaTeX"? I'd say no, since I almost completely leave the TeX world. For example I save the text blocks and reformat them on demand (for example if columns are of different widths or of I need a parshape in the second column).

          – topskip
          Jul 1 '13 at 16:10






        • 16





          I wish I could get feedback for the downvote! Too much advertisement? (Hey, it's open source, so you can use it). Not a good answer? (I think it matches the question.) It would be nice if the downvoter could explain why this answer is a bad answer.

          – topskip
          Jul 2 '13 at 11:13













        • On 'generality' I was thinking for example that there are a few approaches to grid typesetting in TeX, but most (all?) have limitations, often related to display or math mode material. So I was wondering how generally you've been able to deal with this.

          – Joseph Wright
          Jul 2 '13 at 11:36











        • @JosephWright I have an invisible grid in the background with height of the grid cells of baselineskip of the text font. Every object is placed in the grid (and thus integer multiple of baselineskip from the top). There is still much to do but I will for example include an invisible "adjust to grid" marker that goes to the next cell. This part is currently under development.

          – topskip
          Jul 2 '13 at 11:46


















        43














        A shameless plug: We are offering a professional typesetter (though open source) which is based on LuaTeX. If it is "equal or even better" than LaTeX is surely subjective.



        Our focus is to bring the DTP world to automatic typesetting (database publishing). For example we have




        • master pages based on arbitrary conditions

        • text flow based on "frames"

        • multi page tables including subtotals and repeating tablehead and tablefoot

        • easy to install/use fonts

        • grid based layout/typesetting

        • ...


        While TeX / LaTeX is focused on text documents (with or without math) our main application area is product catalogs, price lists and other documents that are created automatically from databases but have flexible and nice layouts.



        http://speedata.github.io/publisher/



        I'd like to invite everyone to try out or perhaps work on the software. Development never ends (as usual).






        share|improve this answer


























        • I'm intrigued by the grid-based support and text flowing. These are both regarded as 'hard' problems with TeX-based systems. How general are your solutions (in broad terms: not looking for anything commercially-sensitive)?

          – Joseph Wright
          Jul 1 '13 at 16:04











        • @JosephWright hard to discuss this in a comment... I believe your question is along the lines: "can we take the approach and throw it at LaTeX"? I'd say no, since I almost completely leave the TeX world. For example I save the text blocks and reformat them on demand (for example if columns are of different widths or of I need a parshape in the second column).

          – topskip
          Jul 1 '13 at 16:10






        • 16





          I wish I could get feedback for the downvote! Too much advertisement? (Hey, it's open source, so you can use it). Not a good answer? (I think it matches the question.) It would be nice if the downvoter could explain why this answer is a bad answer.

          – topskip
          Jul 2 '13 at 11:13













        • On 'generality' I was thinking for example that there are a few approaches to grid typesetting in TeX, but most (all?) have limitations, often related to display or math mode material. So I was wondering how generally you've been able to deal with this.

          – Joseph Wright
          Jul 2 '13 at 11:36











        • @JosephWright I have an invisible grid in the background with height of the grid cells of baselineskip of the text font. Every object is placed in the grid (and thus integer multiple of baselineskip from the top). There is still much to do but I will for example include an invisible "adjust to grid" marker that goes to the next cell. This part is currently under development.

          – topskip
          Jul 2 '13 at 11:46
















        43












        43








        43







        A shameless plug: We are offering a professional typesetter (though open source) which is based on LuaTeX. If it is "equal or even better" than LaTeX is surely subjective.



        Our focus is to bring the DTP world to automatic typesetting (database publishing). For example we have




        • master pages based on arbitrary conditions

        • text flow based on "frames"

        • multi page tables including subtotals and repeating tablehead and tablefoot

        • easy to install/use fonts

        • grid based layout/typesetting

        • ...


        While TeX / LaTeX is focused on text documents (with or without math) our main application area is product catalogs, price lists and other documents that are created automatically from databases but have flexible and nice layouts.



        http://speedata.github.io/publisher/



        I'd like to invite everyone to try out or perhaps work on the software. Development never ends (as usual).






        share|improve this answer















        A shameless plug: We are offering a professional typesetter (though open source) which is based on LuaTeX. If it is "equal or even better" than LaTeX is surely subjective.



        Our focus is to bring the DTP world to automatic typesetting (database publishing). For example we have




        • master pages based on arbitrary conditions

        • text flow based on "frames"

        • multi page tables including subtotals and repeating tablehead and tablefoot

        • easy to install/use fonts

        • grid based layout/typesetting

        • ...


        While TeX / LaTeX is focused on text documents (with or without math) our main application area is product catalogs, price lists and other documents that are created automatically from databases but have flexible and nice layouts.



        http://speedata.github.io/publisher/



        I'd like to invite everyone to try out or perhaps work on the software. Development never ends (as usual).







        share|improve this answer














        share|improve this answer



        share|improve this answer








        edited Jul 2 '13 at 6:48

























        answered Jul 1 '13 at 15:59









        topskiptopskip

        28.5k7116214




        28.5k7116214













        • I'm intrigued by the grid-based support and text flowing. These are both regarded as 'hard' problems with TeX-based systems. How general are your solutions (in broad terms: not looking for anything commercially-sensitive)?

          – Joseph Wright
          Jul 1 '13 at 16:04











        • @JosephWright hard to discuss this in a comment... I believe your question is along the lines: "can we take the approach and throw it at LaTeX"? I'd say no, since I almost completely leave the TeX world. For example I save the text blocks and reformat them on demand (for example if columns are of different widths or of I need a parshape in the second column).

          – topskip
          Jul 1 '13 at 16:10






        • 16





          I wish I could get feedback for the downvote! Too much advertisement? (Hey, it's open source, so you can use it). Not a good answer? (I think it matches the question.) It would be nice if the downvoter could explain why this answer is a bad answer.

          – topskip
          Jul 2 '13 at 11:13













        • On 'generality' I was thinking for example that there are a few approaches to grid typesetting in TeX, but most (all?) have limitations, often related to display or math mode material. So I was wondering how generally you've been able to deal with this.

          – Joseph Wright
          Jul 2 '13 at 11:36











        • @JosephWright I have an invisible grid in the background with height of the grid cells of baselineskip of the text font. Every object is placed in the grid (and thus integer multiple of baselineskip from the top). There is still much to do but I will for example include an invisible "adjust to grid" marker that goes to the next cell. This part is currently under development.

          – topskip
          Jul 2 '13 at 11:46





















        • I'm intrigued by the grid-based support and text flowing. These are both regarded as 'hard' problems with TeX-based systems. How general are your solutions (in broad terms: not looking for anything commercially-sensitive)?

          – Joseph Wright
          Jul 1 '13 at 16:04











        • @JosephWright hard to discuss this in a comment... I believe your question is along the lines: "can we take the approach and throw it at LaTeX"? I'd say no, since I almost completely leave the TeX world. For example I save the text blocks and reformat them on demand (for example if columns are of different widths or of I need a parshape in the second column).

          – topskip
          Jul 1 '13 at 16:10






        • 16





          I wish I could get feedback for the downvote! Too much advertisement? (Hey, it's open source, so you can use it). Not a good answer? (I think it matches the question.) It would be nice if the downvoter could explain why this answer is a bad answer.

          – topskip
          Jul 2 '13 at 11:13













        • On 'generality' I was thinking for example that there are a few approaches to grid typesetting in TeX, but most (all?) have limitations, often related to display or math mode material. So I was wondering how generally you've been able to deal with this.

          – Joseph Wright
          Jul 2 '13 at 11:36











        • @JosephWright I have an invisible grid in the background with height of the grid cells of baselineskip of the text font. Every object is placed in the grid (and thus integer multiple of baselineskip from the top). There is still much to do but I will for example include an invisible "adjust to grid" marker that goes to the next cell. This part is currently under development.

          – topskip
          Jul 2 '13 at 11:46



















        I'm intrigued by the grid-based support and text flowing. These are both regarded as 'hard' problems with TeX-based systems. How general are your solutions (in broad terms: not looking for anything commercially-sensitive)?

        – Joseph Wright
        Jul 1 '13 at 16:04





        I'm intrigued by the grid-based support and text flowing. These are both regarded as 'hard' problems with TeX-based systems. How general are your solutions (in broad terms: not looking for anything commercially-sensitive)?

        – Joseph Wright
        Jul 1 '13 at 16:04













        @JosephWright hard to discuss this in a comment... I believe your question is along the lines: "can we take the approach and throw it at LaTeX"? I'd say no, since I almost completely leave the TeX world. For example I save the text blocks and reformat them on demand (for example if columns are of different widths or of I need a parshape in the second column).

        – topskip
        Jul 1 '13 at 16:10





        @JosephWright hard to discuss this in a comment... I believe your question is along the lines: "can we take the approach and throw it at LaTeX"? I'd say no, since I almost completely leave the TeX world. For example I save the text blocks and reformat them on demand (for example if columns are of different widths or of I need a parshape in the second column).

        – topskip
        Jul 1 '13 at 16:10




        16




        16





        I wish I could get feedback for the downvote! Too much advertisement? (Hey, it's open source, so you can use it). Not a good answer? (I think it matches the question.) It would be nice if the downvoter could explain why this answer is a bad answer.

        – topskip
        Jul 2 '13 at 11:13







        I wish I could get feedback for the downvote! Too much advertisement? (Hey, it's open source, so you can use it). Not a good answer? (I think it matches the question.) It would be nice if the downvoter could explain why this answer is a bad answer.

        – topskip
        Jul 2 '13 at 11:13















        On 'generality' I was thinking for example that there are a few approaches to grid typesetting in TeX, but most (all?) have limitations, often related to display or math mode material. So I was wondering how generally you've been able to deal with this.

        – Joseph Wright
        Jul 2 '13 at 11:36





        On 'generality' I was thinking for example that there are a few approaches to grid typesetting in TeX, but most (all?) have limitations, often related to display or math mode material. So I was wondering how generally you've been able to deal with this.

        – Joseph Wright
        Jul 2 '13 at 11:36













        @JosephWright I have an invisible grid in the background with height of the grid cells of baselineskip of the text font. Every object is placed in the grid (and thus integer multiple of baselineskip from the top). There is still much to do but I will for example include an invisible "adjust to grid" marker that goes to the next cell. This part is currently under development.

        – topskip
        Jul 2 '13 at 11:46







        @JosephWright I have an invisible grid in the background with height of the grid cells of baselineskip of the text font. Every object is placed in the grid (and thus integer multiple of baselineskip from the top). There is still much to do but I will for example include an invisible "adjust to grid" marker that goes to the next cell. This part is currently under development.

        – topskip
        Jul 2 '13 at 11:46













        34














        Troff (http://troff.org/) should definitely be mentioned for its simplicity. I normally use LaTeX for all my documents, but for some special cases (e.g. user manuals) troff (or it's GNU version groff) is just the simpler way. It also can handle math using the preprocessing tool eqn. A nice comparison between troff and LaTeX can be found here.






        share|improve this answer



















        • 1





          I just realised that the link I provided for the comparison actually is a macro package for groff which the author claims makes it comparable to LaTeX. I think I just found something to do for the weekend :)

          – user2426172
          Jun 21 '13 at 8:14






        • 1





          Nice find! Can be a nice alternative to LaTeX for good looking automatically generated documents.

          – gniourf_gniourf
          Jun 21 '13 at 8:26






        • 1





          fyi, from wiki, troff can trace its origins back to a text formatting program called RUNOFF . see RUNOFF en.wikipedia.org/wiki/RUNOFF RUNOFF was the first computer text formatting program to see significant use.

          – Nasser
          Jun 22 '13 at 21:42











        • I used ditroff to typeset my thesis. And I believe groff has taken over some of $TeX$ line-handling and math formatting techniques, so it should be comparable for text at least. But overall a troff document feels like assembler to $LaTeX$' more structured approach. Lerned $'LaTeX$, and never looked back.

          – vonbrand
          Mar 23 '14 at 17:57








        • 1





          +1 for recommending Troff. For sophisticated typesetting, Heirloom Troff is a much better choice than Groff, offering support for microtypography, proper support for OpenType features, improved hyphenation, and - most importantly, native support for Type 1 fonts, TrueType and OpenType. Definitely a formidable contender with TeX for complex, accurate typesetting.

          – Alhadis
          Aug 12 '17 at 17:45
















        34














        Troff (http://troff.org/) should definitely be mentioned for its simplicity. I normally use LaTeX for all my documents, but for some special cases (e.g. user manuals) troff (or it's GNU version groff) is just the simpler way. It also can handle math using the preprocessing tool eqn. A nice comparison between troff and LaTeX can be found here.






        share|improve this answer



















        • 1





          I just realised that the link I provided for the comparison actually is a macro package for groff which the author claims makes it comparable to LaTeX. I think I just found something to do for the weekend :)

          – user2426172
          Jun 21 '13 at 8:14






        • 1





          Nice find! Can be a nice alternative to LaTeX for good looking automatically generated documents.

          – gniourf_gniourf
          Jun 21 '13 at 8:26






        • 1





          fyi, from wiki, troff can trace its origins back to a text formatting program called RUNOFF . see RUNOFF en.wikipedia.org/wiki/RUNOFF RUNOFF was the first computer text formatting program to see significant use.

          – Nasser
          Jun 22 '13 at 21:42











        • I used ditroff to typeset my thesis. And I believe groff has taken over some of $TeX$ line-handling and math formatting techniques, so it should be comparable for text at least. But overall a troff document feels like assembler to $LaTeX$' more structured approach. Lerned $'LaTeX$, and never looked back.

          – vonbrand
          Mar 23 '14 at 17:57








        • 1





          +1 for recommending Troff. For sophisticated typesetting, Heirloom Troff is a much better choice than Groff, offering support for microtypography, proper support for OpenType features, improved hyphenation, and - most importantly, native support for Type 1 fonts, TrueType and OpenType. Definitely a formidable contender with TeX for complex, accurate typesetting.

          – Alhadis
          Aug 12 '17 at 17:45














        34












        34








        34







        Troff (http://troff.org/) should definitely be mentioned for its simplicity. I normally use LaTeX for all my documents, but for some special cases (e.g. user manuals) troff (or it's GNU version groff) is just the simpler way. It also can handle math using the preprocessing tool eqn. A nice comparison between troff and LaTeX can be found here.






        share|improve this answer













        Troff (http://troff.org/) should definitely be mentioned for its simplicity. I normally use LaTeX for all my documents, but for some special cases (e.g. user manuals) troff (or it's GNU version groff) is just the simpler way. It also can handle math using the preprocessing tool eqn. A nice comparison between troff and LaTeX can be found here.







        share|improve this answer












        share|improve this answer



        share|improve this answer










        answered Jun 21 '13 at 8:09









        user2426172user2426172

        44135




        44135








        • 1





          I just realised that the link I provided for the comparison actually is a macro package for groff which the author claims makes it comparable to LaTeX. I think I just found something to do for the weekend :)

          – user2426172
          Jun 21 '13 at 8:14






        • 1





          Nice find! Can be a nice alternative to LaTeX for good looking automatically generated documents.

          – gniourf_gniourf
          Jun 21 '13 at 8:26






        • 1





          fyi, from wiki, troff can trace its origins back to a text formatting program called RUNOFF . see RUNOFF en.wikipedia.org/wiki/RUNOFF RUNOFF was the first computer text formatting program to see significant use.

          – Nasser
          Jun 22 '13 at 21:42











        • I used ditroff to typeset my thesis. And I believe groff has taken over some of $TeX$ line-handling and math formatting techniques, so it should be comparable for text at least. But overall a troff document feels like assembler to $LaTeX$' more structured approach. Lerned $'LaTeX$, and never looked back.

          – vonbrand
          Mar 23 '14 at 17:57








        • 1





          +1 for recommending Troff. For sophisticated typesetting, Heirloom Troff is a much better choice than Groff, offering support for microtypography, proper support for OpenType features, improved hyphenation, and - most importantly, native support for Type 1 fonts, TrueType and OpenType. Definitely a formidable contender with TeX for complex, accurate typesetting.

          – Alhadis
          Aug 12 '17 at 17:45














        • 1





          I just realised that the link I provided for the comparison actually is a macro package for groff which the author claims makes it comparable to LaTeX. I think I just found something to do for the weekend :)

          – user2426172
          Jun 21 '13 at 8:14






        • 1





          Nice find! Can be a nice alternative to LaTeX for good looking automatically generated documents.

          – gniourf_gniourf
          Jun 21 '13 at 8:26






        • 1





          fyi, from wiki, troff can trace its origins back to a text formatting program called RUNOFF . see RUNOFF en.wikipedia.org/wiki/RUNOFF RUNOFF was the first computer text formatting program to see significant use.

          – Nasser
          Jun 22 '13 at 21:42











        • I used ditroff to typeset my thesis. And I believe groff has taken over some of $TeX$ line-handling and math formatting techniques, so it should be comparable for text at least. But overall a troff document feels like assembler to $LaTeX$' more structured approach. Lerned $'LaTeX$, and never looked back.

          – vonbrand
          Mar 23 '14 at 17:57








        • 1





          +1 for recommending Troff. For sophisticated typesetting, Heirloom Troff is a much better choice than Groff, offering support for microtypography, proper support for OpenType features, improved hyphenation, and - most importantly, native support for Type 1 fonts, TrueType and OpenType. Definitely a formidable contender with TeX for complex, accurate typesetting.

          – Alhadis
          Aug 12 '17 at 17:45








        1




        1





        I just realised that the link I provided for the comparison actually is a macro package for groff which the author claims makes it comparable to LaTeX. I think I just found something to do for the weekend :)

        – user2426172
        Jun 21 '13 at 8:14





        I just realised that the link I provided for the comparison actually is a macro package for groff which the author claims makes it comparable to LaTeX. I think I just found something to do for the weekend :)

        – user2426172
        Jun 21 '13 at 8:14




        1




        1





        Nice find! Can be a nice alternative to LaTeX for good looking automatically generated documents.

        – gniourf_gniourf
        Jun 21 '13 at 8:26





        Nice find! Can be a nice alternative to LaTeX for good looking automatically generated documents.

        – gniourf_gniourf
        Jun 21 '13 at 8:26




        1




        1





        fyi, from wiki, troff can trace its origins back to a text formatting program called RUNOFF . see RUNOFF en.wikipedia.org/wiki/RUNOFF RUNOFF was the first computer text formatting program to see significant use.

        – Nasser
        Jun 22 '13 at 21:42





        fyi, from wiki, troff can trace its origins back to a text formatting program called RUNOFF . see RUNOFF en.wikipedia.org/wiki/RUNOFF RUNOFF was the first computer text formatting program to see significant use.

        – Nasser
        Jun 22 '13 at 21:42













        I used ditroff to typeset my thesis. And I believe groff has taken over some of $TeX$ line-handling and math formatting techniques, so it should be comparable for text at least. But overall a troff document feels like assembler to $LaTeX$' more structured approach. Lerned $'LaTeX$, and never looked back.

        – vonbrand
        Mar 23 '14 at 17:57







        I used ditroff to typeset my thesis. And I believe groff has taken over some of $TeX$ line-handling and math formatting techniques, so it should be comparable for text at least. But overall a troff document feels like assembler to $LaTeX$' more structured approach. Lerned $'LaTeX$, and never looked back.

        – vonbrand
        Mar 23 '14 at 17:57






        1




        1





        +1 for recommending Troff. For sophisticated typesetting, Heirloom Troff is a much better choice than Groff, offering support for microtypography, proper support for OpenType features, improved hyphenation, and - most importantly, native support for Type 1 fonts, TrueType and OpenType. Definitely a formidable contender with TeX for complex, accurate typesetting.

        – Alhadis
        Aug 12 '17 at 17:45





        +1 for recommending Troff. For sophisticated typesetting, Heirloom Troff is a much better choice than Groff, offering support for microtypography, proper support for OpenType features, improved hyphenation, and - most importantly, native support for Type 1 fonts, TrueType and OpenType. Definitely a formidable contender with TeX for complex, accurate typesetting.

        – Alhadis
        Aug 12 '17 at 17:45











        33














        GNU TeXMacs is a WYSIWYG scientific structured document editor and typesetter. It used to require a TeX distribution but now it does not.



        Here is a list of not so widely known typesetting systems that I found googling around, mostly work-in-progress (or work-used-to-be-in-progress):




        • cl-typesetting is written in Common Lisp, which implies that its markup language is also Common Lisp. (Macro language, anyone?)


        • The Z format looks like HTML, but is not.


        • Platypus commands look like [list|bullet:{rarrow}]
          . Written in Java.



        • Patoline, written in OCaml. Also handles SQL bibliography databases, and comes with Bibi, a bibfile to SQL converter written in Haskell. (The developers seem to be really keen on functional languages.) You have to build it from source. Commands look like a mixture of TeX, OCaml and new markup:



          (* #FORMAT FormatArticle *)
          begin{genumerate}(AlphaLower, fun s -> [tT (s^". ")])
          item First item
          item Second item
          end{genumerate}
          includeGraphics("pato.png")
          $$ a + b $$
          $ a + b $
          Caml(
          let dr ()=
          [bB (fun _->
          [Drawing (drawing [Path (default,[rectangle (0.,0.) (10.,10.)])])]
          )]
          )


          Note that I have no idea if this code works, I just copied together parts of the manual.




        • SILE (Simon's Improved Layout Engine) is written in Lua with the Pango and Cairo libraries. It also tries to provide a syntax nearly identical to that of LaTeX:



          begin[class=book,papersize=129mm x 198mm]{document}
          include[src=examples/macros.sil]
          script[src=packages/grid]
          left-running-head{A Scandal in Bohemia}
          right-running-head{hfill{}Alexander Conan Doyle}
          headline{A Scandal In Bohemia}

          noindent To Sherlock Holmes she is always em{the woman}.
          footnote{Lorem ipsum dolor sit amet.}

          begin[family=Snell Roundhand]{font}
          Consequetur adipiscing elit.
          end{font}
          end{document}


        • Rinohtype, written in Python. The input format is reStructuredText. See an example input file.



        Maybe more to come.






        share|improve this answer





















        • 1





          AFAIK, TeXmacs was never based on TeX; it uses various TeX algorithms but they are implemented independent of TeX. It uses TeX fonts, though, so that might be why it needed a TeX distribution in the past.

          – Khaled Hosny
          Jul 1 '13 at 22:49













        • Nice edit, I searched for this just to add it :) But it seems that you already added it.

          – Manuel
          Aug 3 '14 at 21:28






        • 2





          This Patoline thing looks promising, in case of active development. It can provide better error messages than TeX/LaTeX, due to types and safety checking.

          – Anton Kochkov
          Aug 19 '14 at 13:16











        • The Patoline manual has not been available for about half a year (patoline.org/patobook.pdf), apparently because of a bug in Patoline, sadly.

          – Benjamin McKay
          Jan 11 '15 at 16:26
















        33














        GNU TeXMacs is a WYSIWYG scientific structured document editor and typesetter. It used to require a TeX distribution but now it does not.



        Here is a list of not so widely known typesetting systems that I found googling around, mostly work-in-progress (or work-used-to-be-in-progress):




        • cl-typesetting is written in Common Lisp, which implies that its markup language is also Common Lisp. (Macro language, anyone?)


        • The Z format looks like HTML, but is not.


        • Platypus commands look like [list|bullet:{rarrow}]
          . Written in Java.



        • Patoline, written in OCaml. Also handles SQL bibliography databases, and comes with Bibi, a bibfile to SQL converter written in Haskell. (The developers seem to be really keen on functional languages.) You have to build it from source. Commands look like a mixture of TeX, OCaml and new markup:



          (* #FORMAT FormatArticle *)
          begin{genumerate}(AlphaLower, fun s -> [tT (s^". ")])
          item First item
          item Second item
          end{genumerate}
          includeGraphics("pato.png")
          $$ a + b $$
          $ a + b $
          Caml(
          let dr ()=
          [bB (fun _->
          [Drawing (drawing [Path (default,[rectangle (0.,0.) (10.,10.)])])]
          )]
          )


          Note that I have no idea if this code works, I just copied together parts of the manual.




        • SILE (Simon's Improved Layout Engine) is written in Lua with the Pango and Cairo libraries. It also tries to provide a syntax nearly identical to that of LaTeX:



          begin[class=book,papersize=129mm x 198mm]{document}
          include[src=examples/macros.sil]
          script[src=packages/grid]
          left-running-head{A Scandal in Bohemia}
          right-running-head{hfill{}Alexander Conan Doyle}
          headline{A Scandal In Bohemia}

          noindent To Sherlock Holmes she is always em{the woman}.
          footnote{Lorem ipsum dolor sit amet.}

          begin[family=Snell Roundhand]{font}
          Consequetur adipiscing elit.
          end{font}
          end{document}


        • Rinohtype, written in Python. The input format is reStructuredText. See an example input file.



        Maybe more to come.






        share|improve this answer





















        • 1





          AFAIK, TeXmacs was never based on TeX; it uses various TeX algorithms but they are implemented independent of TeX. It uses TeX fonts, though, so that might be why it needed a TeX distribution in the past.

          – Khaled Hosny
          Jul 1 '13 at 22:49













        • Nice edit, I searched for this just to add it :) But it seems that you already added it.

          – Manuel
          Aug 3 '14 at 21:28






        • 2





          This Patoline thing looks promising, in case of active development. It can provide better error messages than TeX/LaTeX, due to types and safety checking.

          – Anton Kochkov
          Aug 19 '14 at 13:16











        • The Patoline manual has not been available for about half a year (patoline.org/patobook.pdf), apparently because of a bug in Patoline, sadly.

          – Benjamin McKay
          Jan 11 '15 at 16:26














        33












        33








        33







        GNU TeXMacs is a WYSIWYG scientific structured document editor and typesetter. It used to require a TeX distribution but now it does not.



        Here is a list of not so widely known typesetting systems that I found googling around, mostly work-in-progress (or work-used-to-be-in-progress):




        • cl-typesetting is written in Common Lisp, which implies that its markup language is also Common Lisp. (Macro language, anyone?)


        • The Z format looks like HTML, but is not.


        • Platypus commands look like [list|bullet:{rarrow}]
          . Written in Java.



        • Patoline, written in OCaml. Also handles SQL bibliography databases, and comes with Bibi, a bibfile to SQL converter written in Haskell. (The developers seem to be really keen on functional languages.) You have to build it from source. Commands look like a mixture of TeX, OCaml and new markup:



          (* #FORMAT FormatArticle *)
          begin{genumerate}(AlphaLower, fun s -> [tT (s^". ")])
          item First item
          item Second item
          end{genumerate}
          includeGraphics("pato.png")
          $$ a + b $$
          $ a + b $
          Caml(
          let dr ()=
          [bB (fun _->
          [Drawing (drawing [Path (default,[rectangle (0.,0.) (10.,10.)])])]
          )]
          )


          Note that I have no idea if this code works, I just copied together parts of the manual.




        • SILE (Simon's Improved Layout Engine) is written in Lua with the Pango and Cairo libraries. It also tries to provide a syntax nearly identical to that of LaTeX:



          begin[class=book,papersize=129mm x 198mm]{document}
          include[src=examples/macros.sil]
          script[src=packages/grid]
          left-running-head{A Scandal in Bohemia}
          right-running-head{hfill{}Alexander Conan Doyle}
          headline{A Scandal In Bohemia}

          noindent To Sherlock Holmes she is always em{the woman}.
          footnote{Lorem ipsum dolor sit amet.}

          begin[family=Snell Roundhand]{font}
          Consequetur adipiscing elit.
          end{font}
          end{document}


        • Rinohtype, written in Python. The input format is reStructuredText. See an example input file.



        Maybe more to come.






        share|improve this answer















        GNU TeXMacs is a WYSIWYG scientific structured document editor and typesetter. It used to require a TeX distribution but now it does not.



        Here is a list of not so widely known typesetting systems that I found googling around, mostly work-in-progress (or work-used-to-be-in-progress):




        • cl-typesetting is written in Common Lisp, which implies that its markup language is also Common Lisp. (Macro language, anyone?)


        • The Z format looks like HTML, but is not.


        • Platypus commands look like [list|bullet:{rarrow}]
          . Written in Java.



        • Patoline, written in OCaml. Also handles SQL bibliography databases, and comes with Bibi, a bibfile to SQL converter written in Haskell. (The developers seem to be really keen on functional languages.) You have to build it from source. Commands look like a mixture of TeX, OCaml and new markup:



          (* #FORMAT FormatArticle *)
          begin{genumerate}(AlphaLower, fun s -> [tT (s^". ")])
          item First item
          item Second item
          end{genumerate}
          includeGraphics("pato.png")
          $$ a + b $$
          $ a + b $
          Caml(
          let dr ()=
          [bB (fun _->
          [Drawing (drawing [Path (default,[rectangle (0.,0.) (10.,10.)])])]
          )]
          )


          Note that I have no idea if this code works, I just copied together parts of the manual.




        • SILE (Simon's Improved Layout Engine) is written in Lua with the Pango and Cairo libraries. It also tries to provide a syntax nearly identical to that of LaTeX:



          begin[class=book,papersize=129mm x 198mm]{document}
          include[src=examples/macros.sil]
          script[src=packages/grid]
          left-running-head{A Scandal in Bohemia}
          right-running-head{hfill{}Alexander Conan Doyle}
          headline{A Scandal In Bohemia}

          noindent To Sherlock Holmes she is always em{the woman}.
          footnote{Lorem ipsum dolor sit amet.}

          begin[family=Snell Roundhand]{font}
          Consequetur adipiscing elit.
          end{font}
          end{document}


        • Rinohtype, written in Python. The input format is reStructuredText. See an example input file.



        Maybe more to come.







        share|improve this answer














        share|improve this answer



        share|improve this answer








        edited Mar 9 '18 at 18:25









        larkey

        1327




        1327










        answered Jul 1 '13 at 16:53









        marczellmmarczellm

        7,42153160




        7,42153160








        • 1





          AFAIK, TeXmacs was never based on TeX; it uses various TeX algorithms but they are implemented independent of TeX. It uses TeX fonts, though, so that might be why it needed a TeX distribution in the past.

          – Khaled Hosny
          Jul 1 '13 at 22:49













        • Nice edit, I searched for this just to add it :) But it seems that you already added it.

          – Manuel
          Aug 3 '14 at 21:28






        • 2





          This Patoline thing looks promising, in case of active development. It can provide better error messages than TeX/LaTeX, due to types and safety checking.

          – Anton Kochkov
          Aug 19 '14 at 13:16











        • The Patoline manual has not been available for about half a year (patoline.org/patobook.pdf), apparently because of a bug in Patoline, sadly.

          – Benjamin McKay
          Jan 11 '15 at 16:26














        • 1





          AFAIK, TeXmacs was never based on TeX; it uses various TeX algorithms but they are implemented independent of TeX. It uses TeX fonts, though, so that might be why it needed a TeX distribution in the past.

          – Khaled Hosny
          Jul 1 '13 at 22:49













        • Nice edit, I searched for this just to add it :) But it seems that you already added it.

          – Manuel
          Aug 3 '14 at 21:28






        • 2





          This Patoline thing looks promising, in case of active development. It can provide better error messages than TeX/LaTeX, due to types and safety checking.

          – Anton Kochkov
          Aug 19 '14 at 13:16











        • The Patoline manual has not been available for about half a year (patoline.org/patobook.pdf), apparently because of a bug in Patoline, sadly.

          – Benjamin McKay
          Jan 11 '15 at 16:26








        1




        1





        AFAIK, TeXmacs was never based on TeX; it uses various TeX algorithms but they are implemented independent of TeX. It uses TeX fonts, though, so that might be why it needed a TeX distribution in the past.

        – Khaled Hosny
        Jul 1 '13 at 22:49







        AFAIK, TeXmacs was never based on TeX; it uses various TeX algorithms but they are implemented independent of TeX. It uses TeX fonts, though, so that might be why it needed a TeX distribution in the past.

        – Khaled Hosny
        Jul 1 '13 at 22:49















        Nice edit, I searched for this just to add it :) But it seems that you already added it.

        – Manuel
        Aug 3 '14 at 21:28





        Nice edit, I searched for this just to add it :) But it seems that you already added it.

        – Manuel
        Aug 3 '14 at 21:28




        2




        2





        This Patoline thing looks promising, in case of active development. It can provide better error messages than TeX/LaTeX, due to types and safety checking.

        – Anton Kochkov
        Aug 19 '14 at 13:16





        This Patoline thing looks promising, in case of active development. It can provide better error messages than TeX/LaTeX, due to types and safety checking.

        – Anton Kochkov
        Aug 19 '14 at 13:16













        The Patoline manual has not been available for about half a year (patoline.org/patobook.pdf), apparently because of a bug in Patoline, sadly.

        – Benjamin McKay
        Jan 11 '15 at 16:26





        The Patoline manual has not been available for about half a year (patoline.org/patobook.pdf), apparently because of a bug in Patoline, sadly.

        – Benjamin McKay
        Jan 11 '15 at 16:26











        20














        First, add XyVision to the list of proprietary batch composition competitors.



        Here's a bit of commentary I wrote up a couple of years ago, which still applies. It's also on the ConTeXt wiki.



        https://groups.google.com/forum/message/raw?msg=comp.text.tex/8zugdUxw6dI/mu7Qzs4bQDYJ



        So,




        • using Quark is like being chained to an oar which is covered w/
          splinters and mostly broken at the other end and which will randomly
          break due to being poorly carved (Quark has crashed on me 183 times
          this year) leaving one adrift or run aground, or sometimes returning
          the vessel to its starting point (a few of those crashes have resulted
          in unrecoverable document corruption --- my autobackup folder may
          contain 2 or 3 GBs of files for a given iteration of a particular
          project each month) --- the oar can be smoothed somewhat and reinforced
          (by purchasing or finding XTensions, using XTags &c.) and periodically
          one is required to purchase a new oar (sometimes just after the
          previous one has been customized adequately). For some tasks, one can
          impress any graphic designer as a galley slave to ease the effort for
          others, but while charts are available, there are not automagic
          navigation options and every journey must be manually piloted.


        • using InDesign is pretty much the same except the oar is smoother
          and stronger (it's crashed 29 times on me thus far this year), there
          aren't as many customization options and it's not quite as easy to find
          a candidate for impressment (though soon it'll be as easy as for
          Quark). Charts are available, but again, piloting is strictly manual.


        • using Plain TeX one has to craft the vessel's oar oneself (as well
          as the rest of the vessel unless one is typesetting a clone of The
          TeXbook
          ), but it's as sturdy and as nice a one as one's skills allow
          and can even be an engine which moves the vessel in and of itself ---
          it can be difficult or impossible to find people suitable to help w/
          either carving the oar or using it though, but once a given journey is
          worked out, the oar becomes magical and rows for itself except for when
          one runs into an unplanned for obstacle (the navigation charts are old
          ones and not often up-dated, with a lot of ``terra incognita''),
          allowing one an auto-pilot option for certain journeys, dependent upon
          one's skill.


        • using ePlain, an oar is provided, can be customized, and can be
          enchanted and the charts are okay, but have a lot of ``terra
          incognita'' on them.


        • using LaTeX, an oar is provided and there're lots of nifty
          customizations and improvements already available, and one can impress
          additional oars from CTAN, however on a semi-random basis, adding one
          oar will break other oars, sometimes leaving one adrift or run aground.
          One can enchant a set of oars to accomplish a given journey, easing the
          piloting requirement, and the navigation charts are decent and
          obstacles are fairly well-known.


        • using ConTeXt, a very nice oar is provided, which has lots of
          customization options, but the navigational charts aren't easily read
          by a traditionally trained navigator at first, although they are fairly
          compleat and most journey can be carefully worked out, but once one is,
          it is quite automatic and there's a good auto-pilot option.







        share|improve this answer



















        • 7





          your quotes around terra incognita give away your preference :P

          – percusse
          Jun 28 '13 at 23:33






        • 1





          +1 for style: certainly puts LaTex in context better than the usual table-of-features-I-then-need-to-look-up.

          – WillC
          Aug 29 '18 at 6:32
















        20














        First, add XyVision to the list of proprietary batch composition competitors.



        Here's a bit of commentary I wrote up a couple of years ago, which still applies. It's also on the ConTeXt wiki.



        https://groups.google.com/forum/message/raw?msg=comp.text.tex/8zugdUxw6dI/mu7Qzs4bQDYJ



        So,




        • using Quark is like being chained to an oar which is covered w/
          splinters and mostly broken at the other end and which will randomly
          break due to being poorly carved (Quark has crashed on me 183 times
          this year) leaving one adrift or run aground, or sometimes returning
          the vessel to its starting point (a few of those crashes have resulted
          in unrecoverable document corruption --- my autobackup folder may
          contain 2 or 3 GBs of files for a given iteration of a particular
          project each month) --- the oar can be smoothed somewhat and reinforced
          (by purchasing or finding XTensions, using XTags &c.) and periodically
          one is required to purchase a new oar (sometimes just after the
          previous one has been customized adequately). For some tasks, one can
          impress any graphic designer as a galley slave to ease the effort for
          others, but while charts are available, there are not automagic
          navigation options and every journey must be manually piloted.


        • using InDesign is pretty much the same except the oar is smoother
          and stronger (it's crashed 29 times on me thus far this year), there
          aren't as many customization options and it's not quite as easy to find
          a candidate for impressment (though soon it'll be as easy as for
          Quark). Charts are available, but again, piloting is strictly manual.


        • using Plain TeX one has to craft the vessel's oar oneself (as well
          as the rest of the vessel unless one is typesetting a clone of The
          TeXbook
          ), but it's as sturdy and as nice a one as one's skills allow
          and can even be an engine which moves the vessel in and of itself ---
          it can be difficult or impossible to find people suitable to help w/
          either carving the oar or using it though, but once a given journey is
          worked out, the oar becomes magical and rows for itself except for when
          one runs into an unplanned for obstacle (the navigation charts are old
          ones and not often up-dated, with a lot of ``terra incognita''),
          allowing one an auto-pilot option for certain journeys, dependent upon
          one's skill.


        • using ePlain, an oar is provided, can be customized, and can be
          enchanted and the charts are okay, but have a lot of ``terra
          incognita'' on them.


        • using LaTeX, an oar is provided and there're lots of nifty
          customizations and improvements already available, and one can impress
          additional oars from CTAN, however on a semi-random basis, adding one
          oar will break other oars, sometimes leaving one adrift or run aground.
          One can enchant a set of oars to accomplish a given journey, easing the
          piloting requirement, and the navigation charts are decent and
          obstacles are fairly well-known.


        • using ConTeXt, a very nice oar is provided, which has lots of
          customization options, but the navigational charts aren't easily read
          by a traditionally trained navigator at first, although they are fairly
          compleat and most journey can be carefully worked out, but once one is,
          it is quite automatic and there's a good auto-pilot option.







        share|improve this answer



















        • 7





          your quotes around terra incognita give away your preference :P

          – percusse
          Jun 28 '13 at 23:33






        • 1





          +1 for style: certainly puts LaTex in context better than the usual table-of-features-I-then-need-to-look-up.

          – WillC
          Aug 29 '18 at 6:32














        20












        20








        20







        First, add XyVision to the list of proprietary batch composition competitors.



        Here's a bit of commentary I wrote up a couple of years ago, which still applies. It's also on the ConTeXt wiki.



        https://groups.google.com/forum/message/raw?msg=comp.text.tex/8zugdUxw6dI/mu7Qzs4bQDYJ



        So,




        • using Quark is like being chained to an oar which is covered w/
          splinters and mostly broken at the other end and which will randomly
          break due to being poorly carved (Quark has crashed on me 183 times
          this year) leaving one adrift or run aground, or sometimes returning
          the vessel to its starting point (a few of those crashes have resulted
          in unrecoverable document corruption --- my autobackup folder may
          contain 2 or 3 GBs of files for a given iteration of a particular
          project each month) --- the oar can be smoothed somewhat and reinforced
          (by purchasing or finding XTensions, using XTags &c.) and periodically
          one is required to purchase a new oar (sometimes just after the
          previous one has been customized adequately). For some tasks, one can
          impress any graphic designer as a galley slave to ease the effort for
          others, but while charts are available, there are not automagic
          navigation options and every journey must be manually piloted.


        • using InDesign is pretty much the same except the oar is smoother
          and stronger (it's crashed 29 times on me thus far this year), there
          aren't as many customization options and it's not quite as easy to find
          a candidate for impressment (though soon it'll be as easy as for
          Quark). Charts are available, but again, piloting is strictly manual.


        • using Plain TeX one has to craft the vessel's oar oneself (as well
          as the rest of the vessel unless one is typesetting a clone of The
          TeXbook
          ), but it's as sturdy and as nice a one as one's skills allow
          and can even be an engine which moves the vessel in and of itself ---
          it can be difficult or impossible to find people suitable to help w/
          either carving the oar or using it though, but once a given journey is
          worked out, the oar becomes magical and rows for itself except for when
          one runs into an unplanned for obstacle (the navigation charts are old
          ones and not often up-dated, with a lot of ``terra incognita''),
          allowing one an auto-pilot option for certain journeys, dependent upon
          one's skill.


        • using ePlain, an oar is provided, can be customized, and can be
          enchanted and the charts are okay, but have a lot of ``terra
          incognita'' on them.


        • using LaTeX, an oar is provided and there're lots of nifty
          customizations and improvements already available, and one can impress
          additional oars from CTAN, however on a semi-random basis, adding one
          oar will break other oars, sometimes leaving one adrift or run aground.
          One can enchant a set of oars to accomplish a given journey, easing the
          piloting requirement, and the navigation charts are decent and
          obstacles are fairly well-known.


        • using ConTeXt, a very nice oar is provided, which has lots of
          customization options, but the navigational charts aren't easily read
          by a traditionally trained navigator at first, although they are fairly
          compleat and most journey can be carefully worked out, but once one is,
          it is quite automatic and there's a good auto-pilot option.







        share|improve this answer













        First, add XyVision to the list of proprietary batch composition competitors.



        Here's a bit of commentary I wrote up a couple of years ago, which still applies. It's also on the ConTeXt wiki.



        https://groups.google.com/forum/message/raw?msg=comp.text.tex/8zugdUxw6dI/mu7Qzs4bQDYJ



        So,




        • using Quark is like being chained to an oar which is covered w/
          splinters and mostly broken at the other end and which will randomly
          break due to being poorly carved (Quark has crashed on me 183 times
          this year) leaving one adrift or run aground, or sometimes returning
          the vessel to its starting point (a few of those crashes have resulted
          in unrecoverable document corruption --- my autobackup folder may
          contain 2 or 3 GBs of files for a given iteration of a particular
          project each month) --- the oar can be smoothed somewhat and reinforced
          (by purchasing or finding XTensions, using XTags &c.) and periodically
          one is required to purchase a new oar (sometimes just after the
          previous one has been customized adequately). For some tasks, one can
          impress any graphic designer as a galley slave to ease the effort for
          others, but while charts are available, there are not automagic
          navigation options and every journey must be manually piloted.


        • using InDesign is pretty much the same except the oar is smoother
          and stronger (it's crashed 29 times on me thus far this year), there
          aren't as many customization options and it's not quite as easy to find
          a candidate for impressment (though soon it'll be as easy as for
          Quark). Charts are available, but again, piloting is strictly manual.


        • using Plain TeX one has to craft the vessel's oar oneself (as well
          as the rest of the vessel unless one is typesetting a clone of The
          TeXbook
          ), but it's as sturdy and as nice a one as one's skills allow
          and can even be an engine which moves the vessel in and of itself ---
          it can be difficult or impossible to find people suitable to help w/
          either carving the oar or using it though, but once a given journey is
          worked out, the oar becomes magical and rows for itself except for when
          one runs into an unplanned for obstacle (the navigation charts are old
          ones and not often up-dated, with a lot of ``terra incognita''),
          allowing one an auto-pilot option for certain journeys, dependent upon
          one's skill.


        • using ePlain, an oar is provided, can be customized, and can be
          enchanted and the charts are okay, but have a lot of ``terra
          incognita'' on them.


        • using LaTeX, an oar is provided and there're lots of nifty
          customizations and improvements already available, and one can impress
          additional oars from CTAN, however on a semi-random basis, adding one
          oar will break other oars, sometimes leaving one adrift or run aground.
          One can enchant a set of oars to accomplish a given journey, easing the
          piloting requirement, and the navigation charts are decent and
          obstacles are fairly well-known.


        • using ConTeXt, a very nice oar is provided, which has lots of
          customization options, but the navigational charts aren't easily read
          by a traditionally trained navigator at first, although they are fairly
          compleat and most journey can be carefully worked out, but once one is,
          it is quite automatic and there's a good auto-pilot option.








        share|improve this answer












        share|improve this answer



        share|improve this answer










        answered Jun 28 '13 at 21:02









        WillAdamsWillAdams

        5,5432032




        5,5432032








        • 7





          your quotes around terra incognita give away your preference :P

          – percusse
          Jun 28 '13 at 23:33






        • 1





          +1 for style: certainly puts LaTex in context better than the usual table-of-features-I-then-need-to-look-up.

          – WillC
          Aug 29 '18 at 6:32














        • 7





          your quotes around terra incognita give away your preference :P

          – percusse
          Jun 28 '13 at 23:33






        • 1





          +1 for style: certainly puts LaTex in context better than the usual table-of-features-I-then-need-to-look-up.

          – WillC
          Aug 29 '18 at 6:32








        7




        7





        your quotes around terra incognita give away your preference :P

        – percusse
        Jun 28 '13 at 23:33





        your quotes around terra incognita give away your preference :P

        – percusse
        Jun 28 '13 at 23:33




        1




        1





        +1 for style: certainly puts LaTex in context better than the usual table-of-features-I-then-need-to-look-up.

        – WillC
        Aug 29 '18 at 6:32





        +1 for style: certainly puts LaTex in context better than the usual table-of-features-I-then-need-to-look-up.

        – WillC
        Aug 29 '18 at 6:32











        19














        A new alternative is Patoline, which is still in an early development stage but the goal is to be a modern digital typesetting system.



        As for the LaTeX compatibility one can describe the Patoline compiler as




        Obviously, a related (but much smaller) project is the Patoline compiler, which compiles a mixed Wiki/LaTeX/Ocaml syntax into a variety of output formats, including traditional PDF files, but also web servers that deliver dynamic contents, synchronized for instance with a talk.




        The documentation includes a PhD thesis and a description of the syntax which was produced using Patoline, and which looks to me as an amateur in typesetting pretty good:



        enter image description here






        share|improve this answer


























        • However, Patoline has many flaws that make it very hard to use at the moment. As well, it introduces some concepts that I don't quite like. I wonder if Xavier and his collaborators manage to make it as tweakable as TeX is, in the sense that any feature it has has to be tunable, and in particular, easy to disable.

          – yo'
          Aug 19 '14 at 14:03






        • 1





          @tohecz: I have unfortunately not found the time to try it out myself. Still, I am hoping for an alternative to LaTeX which keeps the good features but replaces the 'programming language' with something more reasonable. LuaTeX is definitely a step in the right direction.

          – Alexander
          Aug 20 '14 at 13:37






        • 2





          That's another point: OCaml doesn't look like a good choice...

          – yo'
          Aug 20 '14 at 14:08













        • sorry I used the wrong name in the 1st comment, the main developer is of course Pierre-Etienne, not Xavier.

          – yo'
          Aug 20 '14 at 14:13
















        19














        A new alternative is Patoline, which is still in an early development stage but the goal is to be a modern digital typesetting system.



        As for the LaTeX compatibility one can describe the Patoline compiler as




        Obviously, a related (but much smaller) project is the Patoline compiler, which compiles a mixed Wiki/LaTeX/Ocaml syntax into a variety of output formats, including traditional PDF files, but also web servers that deliver dynamic contents, synchronized for instance with a talk.




        The documentation includes a PhD thesis and a description of the syntax which was produced using Patoline, and which looks to me as an amateur in typesetting pretty good:



        enter image description here






        share|improve this answer


























        • However, Patoline has many flaws that make it very hard to use at the moment. As well, it introduces some concepts that I don't quite like. I wonder if Xavier and his collaborators manage to make it as tweakable as TeX is, in the sense that any feature it has has to be tunable, and in particular, easy to disable.

          – yo'
          Aug 19 '14 at 14:03






        • 1





          @tohecz: I have unfortunately not found the time to try it out myself. Still, I am hoping for an alternative to LaTeX which keeps the good features but replaces the 'programming language' with something more reasonable. LuaTeX is definitely a step in the right direction.

          – Alexander
          Aug 20 '14 at 13:37






        • 2





          That's another point: OCaml doesn't look like a good choice...

          – yo'
          Aug 20 '14 at 14:08













        • sorry I used the wrong name in the 1st comment, the main developer is of course Pierre-Etienne, not Xavier.

          – yo'
          Aug 20 '14 at 14:13














        19












        19








        19







        A new alternative is Patoline, which is still in an early development stage but the goal is to be a modern digital typesetting system.



        As for the LaTeX compatibility one can describe the Patoline compiler as




        Obviously, a related (but much smaller) project is the Patoline compiler, which compiles a mixed Wiki/LaTeX/Ocaml syntax into a variety of output formats, including traditional PDF files, but also web servers that deliver dynamic contents, synchronized for instance with a talk.




        The documentation includes a PhD thesis and a description of the syntax which was produced using Patoline, and which looks to me as an amateur in typesetting pretty good:



        enter image description here






        share|improve this answer















        A new alternative is Patoline, which is still in an early development stage but the goal is to be a modern digital typesetting system.



        As for the LaTeX compatibility one can describe the Patoline compiler as




        Obviously, a related (but much smaller) project is the Patoline compiler, which compiles a mixed Wiki/LaTeX/Ocaml syntax into a variety of output formats, including traditional PDF files, but also web servers that deliver dynamic contents, synchronized for instance with a talk.




        The documentation includes a PhD thesis and a description of the syntax which was produced using Patoline, and which looks to me as an amateur in typesetting pretty good:



        enter image description here







        share|improve this answer














        share|improve this answer



        share|improve this answer








        edited Mar 9 '18 at 18:52









        larkey

        1327




        1327










        answered Aug 19 '14 at 13:47









        AlexanderAlexander

        5,69132350




        5,69132350













        • However, Patoline has many flaws that make it very hard to use at the moment. As well, it introduces some concepts that I don't quite like. I wonder if Xavier and his collaborators manage to make it as tweakable as TeX is, in the sense that any feature it has has to be tunable, and in particular, easy to disable.

          – yo'
          Aug 19 '14 at 14:03






        • 1





          @tohecz: I have unfortunately not found the time to try it out myself. Still, I am hoping for an alternative to LaTeX which keeps the good features but replaces the 'programming language' with something more reasonable. LuaTeX is definitely a step in the right direction.

          – Alexander
          Aug 20 '14 at 13:37






        • 2





          That's another point: OCaml doesn't look like a good choice...

          – yo'
          Aug 20 '14 at 14:08













        • sorry I used the wrong name in the 1st comment, the main developer is of course Pierre-Etienne, not Xavier.

          – yo'
          Aug 20 '14 at 14:13



















        • However, Patoline has many flaws that make it very hard to use at the moment. As well, it introduces some concepts that I don't quite like. I wonder if Xavier and his collaborators manage to make it as tweakable as TeX is, in the sense that any feature it has has to be tunable, and in particular, easy to disable.

          – yo'
          Aug 19 '14 at 14:03






        • 1





          @tohecz: I have unfortunately not found the time to try it out myself. Still, I am hoping for an alternative to LaTeX which keeps the good features but replaces the 'programming language' with something more reasonable. LuaTeX is definitely a step in the right direction.

          – Alexander
          Aug 20 '14 at 13:37






        • 2





          That's another point: OCaml doesn't look like a good choice...

          – yo'
          Aug 20 '14 at 14:08













        • sorry I used the wrong name in the 1st comment, the main developer is of course Pierre-Etienne, not Xavier.

          – yo'
          Aug 20 '14 at 14:13

















        However, Patoline has many flaws that make it very hard to use at the moment. As well, it introduces some concepts that I don't quite like. I wonder if Xavier and his collaborators manage to make it as tweakable as TeX is, in the sense that any feature it has has to be tunable, and in particular, easy to disable.

        – yo'
        Aug 19 '14 at 14:03





        However, Patoline has many flaws that make it very hard to use at the moment. As well, it introduces some concepts that I don't quite like. I wonder if Xavier and his collaborators manage to make it as tweakable as TeX is, in the sense that any feature it has has to be tunable, and in particular, easy to disable.

        – yo'
        Aug 19 '14 at 14:03




        1




        1





        @tohecz: I have unfortunately not found the time to try it out myself. Still, I am hoping for an alternative to LaTeX which keeps the good features but replaces the 'programming language' with something more reasonable. LuaTeX is definitely a step in the right direction.

        – Alexander
        Aug 20 '14 at 13:37





        @tohecz: I have unfortunately not found the time to try it out myself. Still, I am hoping for an alternative to LaTeX which keeps the good features but replaces the 'programming language' with something more reasonable. LuaTeX is definitely a step in the right direction.

        – Alexander
        Aug 20 '14 at 13:37




        2




        2





        That's another point: OCaml doesn't look like a good choice...

        – yo'
        Aug 20 '14 at 14:08







        That's another point: OCaml doesn't look like a good choice...

        – yo'
        Aug 20 '14 at 14:08















        sorry I used the wrong name in the 1st comment, the main developer is of course Pierre-Etienne, not Xavier.

        – yo'
        Aug 20 '14 at 14:13





        sorry I used the wrong name in the 1st comment, the main developer is of course Pierre-Etienne, not Xavier.

        – yo'
        Aug 20 '14 at 14:13











        16














        While LaTeX is the markup language of choice in many scientific and scholarly circles (especially in the fields of mathematics, computer science, physics, and astronomy), some scholars are starting to write papers using new markup languages tailored for the web. Markdown is one of them. It is very versatile and it can be easily extended to satisfy the needs of scientific writing. The advantage of using Markdown (or similar) instead of LaTeX is that we are writing on the web more than ever. Thus, it makes sense to use a web native typesetting format. For those who would like to use both Markdown and Latex on the web, Authorea (full disclosure, I am a co-founder) is an online social authoring platform that allows articles to be edited in both formats simultaneously. Authorea also renders and compiles LaTeX and MathML equations to the web, in addition to PDF.






        share|improve this answer





















        • 3





          LaTeX is not only markup but also typesetting. And welcome to TeX.sx!

          – Qrrbrbirlbel
          Jun 25 '13 at 22:56











        • Welcome to TeX.sx! That platform looks quite intriguing, indeed. Do you have any relationship to the company that runs it (apart from being a user)?

          – Jake
          Jun 26 '13 at 1:31











        • hi Jake- yes I am the co-founder of Authorea (and a user, since I still write papers in Astronomy + Information Science). We have a couple of thousand users and the vast majority of them writes in LaTeX.

          – Alberto Pepe
          Jun 26 '13 at 15:44











        • @user32799: Thanks for the reply. When posting an answer promoting a product you're affiliated with, that affiliation should be disclosed (see tex.stackexchange.com/help/behavior). Could you edit your post to include that information?

          – Jake
          Jun 28 '13 at 20:50











        • hi Jake- I just updated my .sx profile information with my affiliation. Thanks!

          – Alberto Pepe
          Jun 29 '13 at 21:02
















        16














        While LaTeX is the markup language of choice in many scientific and scholarly circles (especially in the fields of mathematics, computer science, physics, and astronomy), some scholars are starting to write papers using new markup languages tailored for the web. Markdown is one of them. It is very versatile and it can be easily extended to satisfy the needs of scientific writing. The advantage of using Markdown (or similar) instead of LaTeX is that we are writing on the web more than ever. Thus, it makes sense to use a web native typesetting format. For those who would like to use both Markdown and Latex on the web, Authorea (full disclosure, I am a co-founder) is an online social authoring platform that allows articles to be edited in both formats simultaneously. Authorea also renders and compiles LaTeX and MathML equations to the web, in addition to PDF.






        share|improve this answer





















        • 3





          LaTeX is not only markup but also typesetting. And welcome to TeX.sx!

          – Qrrbrbirlbel
          Jun 25 '13 at 22:56











        • Welcome to TeX.sx! That platform looks quite intriguing, indeed. Do you have any relationship to the company that runs it (apart from being a user)?

          – Jake
          Jun 26 '13 at 1:31











        • hi Jake- yes I am the co-founder of Authorea (and a user, since I still write papers in Astronomy + Information Science). We have a couple of thousand users and the vast majority of them writes in LaTeX.

          – Alberto Pepe
          Jun 26 '13 at 15:44











        • @user32799: Thanks for the reply. When posting an answer promoting a product you're affiliated with, that affiliation should be disclosed (see tex.stackexchange.com/help/behavior). Could you edit your post to include that information?

          – Jake
          Jun 28 '13 at 20:50











        • hi Jake- I just updated my .sx profile information with my affiliation. Thanks!

          – Alberto Pepe
          Jun 29 '13 at 21:02














        16












        16








        16







        While LaTeX is the markup language of choice in many scientific and scholarly circles (especially in the fields of mathematics, computer science, physics, and astronomy), some scholars are starting to write papers using new markup languages tailored for the web. Markdown is one of them. It is very versatile and it can be easily extended to satisfy the needs of scientific writing. The advantage of using Markdown (or similar) instead of LaTeX is that we are writing on the web more than ever. Thus, it makes sense to use a web native typesetting format. For those who would like to use both Markdown and Latex on the web, Authorea (full disclosure, I am a co-founder) is an online social authoring platform that allows articles to be edited in both formats simultaneously. Authorea also renders and compiles LaTeX and MathML equations to the web, in addition to PDF.






        share|improve this answer















        While LaTeX is the markup language of choice in many scientific and scholarly circles (especially in the fields of mathematics, computer science, physics, and astronomy), some scholars are starting to write papers using new markup languages tailored for the web. Markdown is one of them. It is very versatile and it can be easily extended to satisfy the needs of scientific writing. The advantage of using Markdown (or similar) instead of LaTeX is that we are writing on the web more than ever. Thus, it makes sense to use a web native typesetting format. For those who would like to use both Markdown and Latex on the web, Authorea (full disclosure, I am a co-founder) is an online social authoring platform that allows articles to be edited in both formats simultaneously. Authorea also renders and compiles LaTeX and MathML equations to the web, in addition to PDF.







        share|improve this answer














        share|improve this answer



        share|improve this answer








        edited Jul 1 '13 at 14:15

























        answered Jun 25 '13 at 22:33









        Alberto PepeAlberto Pepe

        56469




        56469








        • 3





          LaTeX is not only markup but also typesetting. And welcome to TeX.sx!

          – Qrrbrbirlbel
          Jun 25 '13 at 22:56











        • Welcome to TeX.sx! That platform looks quite intriguing, indeed. Do you have any relationship to the company that runs it (apart from being a user)?

          – Jake
          Jun 26 '13 at 1:31











        • hi Jake- yes I am the co-founder of Authorea (and a user, since I still write papers in Astronomy + Information Science). We have a couple of thousand users and the vast majority of them writes in LaTeX.

          – Alberto Pepe
          Jun 26 '13 at 15:44











        • @user32799: Thanks for the reply. When posting an answer promoting a product you're affiliated with, that affiliation should be disclosed (see tex.stackexchange.com/help/behavior). Could you edit your post to include that information?

          – Jake
          Jun 28 '13 at 20:50











        • hi Jake- I just updated my .sx profile information with my affiliation. Thanks!

          – Alberto Pepe
          Jun 29 '13 at 21:02














        • 3





          LaTeX is not only markup but also typesetting. And welcome to TeX.sx!

          – Qrrbrbirlbel
          Jun 25 '13 at 22:56











        • Welcome to TeX.sx! That platform looks quite intriguing, indeed. Do you have any relationship to the company that runs it (apart from being a user)?

          – Jake
          Jun 26 '13 at 1:31











        • hi Jake- yes I am the co-founder of Authorea (and a user, since I still write papers in Astronomy + Information Science). We have a couple of thousand users and the vast majority of them writes in LaTeX.

          – Alberto Pepe
          Jun 26 '13 at 15:44











        • @user32799: Thanks for the reply. When posting an answer promoting a product you're affiliated with, that affiliation should be disclosed (see tex.stackexchange.com/help/behavior). Could you edit your post to include that information?

          – Jake
          Jun 28 '13 at 20:50











        • hi Jake- I just updated my .sx profile information with my affiliation. Thanks!

          – Alberto Pepe
          Jun 29 '13 at 21:02








        3




        3





        LaTeX is not only markup but also typesetting. And welcome to TeX.sx!

        – Qrrbrbirlbel
        Jun 25 '13 at 22:56





        LaTeX is not only markup but also typesetting. And welcome to TeX.sx!

        – Qrrbrbirlbel
        Jun 25 '13 at 22:56













        Welcome to TeX.sx! That platform looks quite intriguing, indeed. Do you have any relationship to the company that runs it (apart from being a user)?

        – Jake
        Jun 26 '13 at 1:31





        Welcome to TeX.sx! That platform looks quite intriguing, indeed. Do you have any relationship to the company that runs it (apart from being a user)?

        – Jake
        Jun 26 '13 at 1:31













        hi Jake- yes I am the co-founder of Authorea (and a user, since I still write papers in Astronomy + Information Science). We have a couple of thousand users and the vast majority of them writes in LaTeX.

        – Alberto Pepe
        Jun 26 '13 at 15:44





        hi Jake- yes I am the co-founder of Authorea (and a user, since I still write papers in Astronomy + Information Science). We have a couple of thousand users and the vast majority of them writes in LaTeX.

        – Alberto Pepe
        Jun 26 '13 at 15:44













        @user32799: Thanks for the reply. When posting an answer promoting a product you're affiliated with, that affiliation should be disclosed (see tex.stackexchange.com/help/behavior). Could you edit your post to include that information?

        – Jake
        Jun 28 '13 at 20:50





        @user32799: Thanks for the reply. When posting an answer promoting a product you're affiliated with, that affiliation should be disclosed (see tex.stackexchange.com/help/behavior). Could you edit your post to include that information?

        – Jake
        Jun 28 '13 at 20:50













        hi Jake- I just updated my .sx profile information with my affiliation. Thanks!

        – Alberto Pepe
        Jun 29 '13 at 21:02





        hi Jake- I just updated my .sx profile information with my affiliation. Thanks!

        – Alberto Pepe
        Jun 29 '13 at 21:02











        14














        TeX and friends maybe still the best. There are some alternative and results are various. I do not really use any of them, but kind of know there existence.




        • Lout

        • ANT






        share|improve this answer



















        • 2





          Another one: asciidoc

          – qunying
          Jun 20 '13 at 23:34








        • 2





          there was also chi-writer. no longer around. en.wikipedia.org/wiki/ChiWriter

          – Nasser
          Jun 20 '13 at 23:44






        • 3





          ANT is an impressive codebase, especially the support they have for OTF. Also, the embedded scripting languages is a huge win. Unfortunately it depends on archaic versions of libraries and the language, so it won’t build on current systems. The most recent post on their mailing list is from 2010 so I guess it could be considered unmaintained.

          – Philipp Gesang
          Jun 21 '13 at 9:10






        • 3





          I'm really interested in whether Lout and Ant have ever been used. Can you give examples/sample output if there's any? (Apart from their respective user manuals, of course.) I'm also interested in their "stories", why they are unmaintained, what happened with the author, etc.

          – marczellm
          Jun 22 '13 at 8:37











        • Follow the example in Lout's user manual will do. Lout is still under development, it just released 3.40. Jeffrey (the author) does not use source version control. But you could always look at the mailing list, it is a low volume list but active. He plan his next typesetting engine Nonpareil, but seems not much news yet. Main drawback that Lout doesn't support unicode. Not sure about ANT.

          – qunying
          Jun 26 '13 at 17:23
















        14














        TeX and friends maybe still the best. There are some alternative and results are various. I do not really use any of them, but kind of know there existence.




        • Lout

        • ANT






        share|improve this answer



















        • 2





          Another one: asciidoc

          – qunying
          Jun 20 '13 at 23:34








        • 2





          there was also chi-writer. no longer around. en.wikipedia.org/wiki/ChiWriter

          – Nasser
          Jun 20 '13 at 23:44






        • 3





          ANT is an impressive codebase, especially the support they have for OTF. Also, the embedded scripting languages is a huge win. Unfortunately it depends on archaic versions of libraries and the language, so it won’t build on current systems. The most recent post on their mailing list is from 2010 so I guess it could be considered unmaintained.

          – Philipp Gesang
          Jun 21 '13 at 9:10






        • 3





          I'm really interested in whether Lout and Ant have ever been used. Can you give examples/sample output if there's any? (Apart from their respective user manuals, of course.) I'm also interested in their "stories", why they are unmaintained, what happened with the author, etc.

          – marczellm
          Jun 22 '13 at 8:37











        • Follow the example in Lout's user manual will do. Lout is still under development, it just released 3.40. Jeffrey (the author) does not use source version control. But you could always look at the mailing list, it is a low volume list but active. He plan his next typesetting engine Nonpareil, but seems not much news yet. Main drawback that Lout doesn't support unicode. Not sure about ANT.

          – qunying
          Jun 26 '13 at 17:23














        14












        14








        14







        TeX and friends maybe still the best. There are some alternative and results are various. I do not really use any of them, but kind of know there existence.




        • Lout

        • ANT






        share|improve this answer













        TeX and friends maybe still the best. There are some alternative and results are various. I do not really use any of them, but kind of know there existence.




        • Lout

        • ANT







        share|improve this answer












        share|improve this answer



        share|improve this answer










        answered Jun 20 '13 at 23:33









        qunyingqunying

        33623




        33623








        • 2





          Another one: asciidoc

          – qunying
          Jun 20 '13 at 23:34








        • 2





          there was also chi-writer. no longer around. en.wikipedia.org/wiki/ChiWriter

          – Nasser
          Jun 20 '13 at 23:44






        • 3





          ANT is an impressive codebase, especially the support they have for OTF. Also, the embedded scripting languages is a huge win. Unfortunately it depends on archaic versions of libraries and the language, so it won’t build on current systems. The most recent post on their mailing list is from 2010 so I guess it could be considered unmaintained.

          – Philipp Gesang
          Jun 21 '13 at 9:10






        • 3





          I'm really interested in whether Lout and Ant have ever been used. Can you give examples/sample output if there's any? (Apart from their respective user manuals, of course.) I'm also interested in their "stories", why they are unmaintained, what happened with the author, etc.

          – marczellm
          Jun 22 '13 at 8:37











        • Follow the example in Lout's user manual will do. Lout is still under development, it just released 3.40. Jeffrey (the author) does not use source version control. But you could always look at the mailing list, it is a low volume list but active. He plan his next typesetting engine Nonpareil, but seems not much news yet. Main drawback that Lout doesn't support unicode. Not sure about ANT.

          – qunying
          Jun 26 '13 at 17:23














        • 2





          Another one: asciidoc

          – qunying
          Jun 20 '13 at 23:34








        • 2





          there was also chi-writer. no longer around. en.wikipedia.org/wiki/ChiWriter

          – Nasser
          Jun 20 '13 at 23:44






        • 3





          ANT is an impressive codebase, especially the support they have for OTF. Also, the embedded scripting languages is a huge win. Unfortunately it depends on archaic versions of libraries and the language, so it won’t build on current systems. The most recent post on their mailing list is from 2010 so I guess it could be considered unmaintained.

          – Philipp Gesang
          Jun 21 '13 at 9:10






        • 3





          I'm really interested in whether Lout and Ant have ever been used. Can you give examples/sample output if there's any? (Apart from their respective user manuals, of course.) I'm also interested in their "stories", why they are unmaintained, what happened with the author, etc.

          – marczellm
          Jun 22 '13 at 8:37











        • Follow the example in Lout's user manual will do. Lout is still under development, it just released 3.40. Jeffrey (the author) does not use source version control. But you could always look at the mailing list, it is a low volume list but active. He plan his next typesetting engine Nonpareil, but seems not much news yet. Main drawback that Lout doesn't support unicode. Not sure about ANT.

          – qunying
          Jun 26 '13 at 17:23








        2




        2





        Another one: asciidoc

        – qunying
        Jun 20 '13 at 23:34







        Another one: asciidoc

        – qunying
        Jun 20 '13 at 23:34






        2




        2





        there was also chi-writer. no longer around. en.wikipedia.org/wiki/ChiWriter

        – Nasser
        Jun 20 '13 at 23:44





        there was also chi-writer. no longer around. en.wikipedia.org/wiki/ChiWriter

        – Nasser
        Jun 20 '13 at 23:44




        3




        3





        ANT is an impressive codebase, especially the support they have for OTF. Also, the embedded scripting languages is a huge win. Unfortunately it depends on archaic versions of libraries and the language, so it won’t build on current systems. The most recent post on their mailing list is from 2010 so I guess it could be considered unmaintained.

        – Philipp Gesang
        Jun 21 '13 at 9:10





        ANT is an impressive codebase, especially the support they have for OTF. Also, the embedded scripting languages is a huge win. Unfortunately it depends on archaic versions of libraries and the language, so it won’t build on current systems. The most recent post on their mailing list is from 2010 so I guess it could be considered unmaintained.

        – Philipp Gesang
        Jun 21 '13 at 9:10




        3




        3





        I'm really interested in whether Lout and Ant have ever been used. Can you give examples/sample output if there's any? (Apart from their respective user manuals, of course.) I'm also interested in their "stories", why they are unmaintained, what happened with the author, etc.

        – marczellm
        Jun 22 '13 at 8:37





        I'm really interested in whether Lout and Ant have ever been used. Can you give examples/sample output if there's any? (Apart from their respective user manuals, of course.) I'm also interested in their "stories", why they are unmaintained, what happened with the author, etc.

        – marczellm
        Jun 22 '13 at 8:37













        Follow the example in Lout's user manual will do. Lout is still under development, it just released 3.40. Jeffrey (the author) does not use source version control. But you could always look at the mailing list, it is a low volume list but active. He plan his next typesetting engine Nonpareil, but seems not much news yet. Main drawback that Lout doesn't support unicode. Not sure about ANT.

        – qunying
        Jun 26 '13 at 17:23





        Follow the example in Lout's user manual will do. Lout is still under development, it just released 3.40. Jeffrey (the author) does not use source version control. But you could always look at the mailing list, it is a low volume list but active. He plan his next typesetting engine Nonpareil, but seems not much news yet. Main drawback that Lout doesn't support unicode. Not sure about ANT.

        – qunying
        Jun 26 '13 at 17:23











        10














        As long as you don't need mathematical typesetting, you actually can find better than TeX with Heirloom Documentation Tools. Not only does it provide Knuth's algorithm for formatting paragraphs; it also allows to compute spacing by mixing three systems (interletters spacing, interwords spacing, imperceptible change in the shapes of the glyphs). Thus you can work with constant space between words! Selecting a font among various kinds of fonts is much easier and quicker than with TeX. I have been using it for years while my colleagues use LaTeX; they wouldn't have the idea to only think their documents can be on a par with mine. If you are interested in typography, then Heirloom version of troff is definitely the way to do.



        edit: Since I first wrote this answer, it looks like new pieces of software have appeared. I just heard about utroff which seems to come from Heirloom troff; I will very certainly have a look at it and probably use it now because Heirloom troff doesn't seem to be actively maintained and may be difficult to compile.






        share|improve this answer






























          10














          As long as you don't need mathematical typesetting, you actually can find better than TeX with Heirloom Documentation Tools. Not only does it provide Knuth's algorithm for formatting paragraphs; it also allows to compute spacing by mixing three systems (interletters spacing, interwords spacing, imperceptible change in the shapes of the glyphs). Thus you can work with constant space between words! Selecting a font among various kinds of fonts is much easier and quicker than with TeX. I have been using it for years while my colleagues use LaTeX; they wouldn't have the idea to only think their documents can be on a par with mine. If you are interested in typography, then Heirloom version of troff is definitely the way to do.



          edit: Since I first wrote this answer, it looks like new pieces of software have appeared. I just heard about utroff which seems to come from Heirloom troff; I will very certainly have a look at it and probably use it now because Heirloom troff doesn't seem to be actively maintained and may be difficult to compile.






          share|improve this answer




























            10












            10








            10







            As long as you don't need mathematical typesetting, you actually can find better than TeX with Heirloom Documentation Tools. Not only does it provide Knuth's algorithm for formatting paragraphs; it also allows to compute spacing by mixing three systems (interletters spacing, interwords spacing, imperceptible change in the shapes of the glyphs). Thus you can work with constant space between words! Selecting a font among various kinds of fonts is much easier and quicker than with TeX. I have been using it for years while my colleagues use LaTeX; they wouldn't have the idea to only think their documents can be on a par with mine. If you are interested in typography, then Heirloom version of troff is definitely the way to do.



            edit: Since I first wrote this answer, it looks like new pieces of software have appeared. I just heard about utroff which seems to come from Heirloom troff; I will very certainly have a look at it and probably use it now because Heirloom troff doesn't seem to be actively maintained and may be difficult to compile.






            share|improve this answer















            As long as you don't need mathematical typesetting, you actually can find better than TeX with Heirloom Documentation Tools. Not only does it provide Knuth's algorithm for formatting paragraphs; it also allows to compute spacing by mixing three systems (interletters spacing, interwords spacing, imperceptible change in the shapes of the glyphs). Thus you can work with constant space between words! Selecting a font among various kinds of fonts is much easier and quicker than with TeX. I have been using it for years while my colleagues use LaTeX; they wouldn't have the idea to only think their documents can be on a par with mine. If you are interested in typography, then Heirloom version of troff is definitely the way to do.



            edit: Since I first wrote this answer, it looks like new pieces of software have appeared. I just heard about utroff which seems to come from Heirloom troff; I will very certainly have a look at it and probably use it now because Heirloom troff doesn't seem to be actively maintained and may be difficult to compile.







            share|improve this answer














            share|improve this answer



            share|improve this answer








            edited Nov 14 '14 at 19:17

























            answered Mar 23 '14 at 15:40









            Thomas BaruchelThomas Baruchel

            22926




            22926























                1














                Scientific Word can be another alternative software for scientists:
                http://www.mackichan.com/index.html?products/sw.html~mainFrame






                share|improve this answer





















                • 3





                  Scientific Word comprises a TeX distribution along with a quasi-WYSIWYG front end. The code it generates is (mostly) LaTeX. I'm thus doubtful that Scientific Word can be considered a alternative to LaTeX. Separately, the software hasn't seen an upgrade in years and years. The current version, numbered 5.5, has been around since at least 2007 (and probably even longer). A software package that's been around essentially unchanged for more than seven years is functionally obsolete, I'm afraid.

                  – Mico
                  Jan 11 '15 at 17:22











                • @Mico As far as I agree, the same thing can essentially be said about TeX.

                  – Gaussler
                  Jul 11 '15 at 14:27











                • @Gaussler - Version 6 of Scientific Word has been released quite recently; I'm sure the long-suffering users of SW were jubilant. I don't agree that SW's situation is comparable to that of TeX. (i) TeXLive has been every year for many years now. (ii) While "Plain TeX" is frozen, that's not the case for pdfLaTeX, XeTeX, LuaTeX, and lots and lots of packages for pdfLaTeX, XeLaTeX, and LuaTeX. (iii) LaTeX2e was also nearly frozen for many years, but got a significant update as of 1/1/2015.

                  – Mico
                  Jul 11 '15 at 14:54













                • @Mico Still, the pdfTeX backend has remained dominant and frozen since forever. Also, you compare Plain TeX - a format - to LuaTeX and XeTeX - two engines.

                  – Gaussler
                  Jul 11 '15 at 15:01








                • 1





                  @Gaussler - I'm no good at linguistic quibbles. Can't vouch for what "standard" terminology may be. All I can hope for is that I've clarified what I meant to say.

                  – Mico
                  Jul 11 '15 at 15:25


















                1














                Scientific Word can be another alternative software for scientists:
                http://www.mackichan.com/index.html?products/sw.html~mainFrame






                share|improve this answer





















                • 3





                  Scientific Word comprises a TeX distribution along with a quasi-WYSIWYG front end. The code it generates is (mostly) LaTeX. I'm thus doubtful that Scientific Word can be considered a alternative to LaTeX. Separately, the software hasn't seen an upgrade in years and years. The current version, numbered 5.5, has been around since at least 2007 (and probably even longer). A software package that's been around essentially unchanged for more than seven years is functionally obsolete, I'm afraid.

                  – Mico
                  Jan 11 '15 at 17:22











                • @Mico As far as I agree, the same thing can essentially be said about TeX.

                  – Gaussler
                  Jul 11 '15 at 14:27











                • @Gaussler - Version 6 of Scientific Word has been released quite recently; I'm sure the long-suffering users of SW were jubilant. I don't agree that SW's situation is comparable to that of TeX. (i) TeXLive has been every year for many years now. (ii) While "Plain TeX" is frozen, that's not the case for pdfLaTeX, XeTeX, LuaTeX, and lots and lots of packages for pdfLaTeX, XeLaTeX, and LuaTeX. (iii) LaTeX2e was also nearly frozen for many years, but got a significant update as of 1/1/2015.

                  – Mico
                  Jul 11 '15 at 14:54













                • @Mico Still, the pdfTeX backend has remained dominant and frozen since forever. Also, you compare Plain TeX - a format - to LuaTeX and XeTeX - two engines.

                  – Gaussler
                  Jul 11 '15 at 15:01








                • 1





                  @Gaussler - I'm no good at linguistic quibbles. Can't vouch for what "standard" terminology may be. All I can hope for is that I've clarified what I meant to say.

                  – Mico
                  Jul 11 '15 at 15:25
















                1












                1








                1







                Scientific Word can be another alternative software for scientists:
                http://www.mackichan.com/index.html?products/sw.html~mainFrame






                share|improve this answer















                Scientific Word can be another alternative software for scientists:
                http://www.mackichan.com/index.html?products/sw.html~mainFrame







                share|improve this answer














                share|improve this answer



                share|improve this answer








                edited Aug 4 '14 at 21:57

























                answered Aug 4 '14 at 11:10









                ux123ux123

                344




                344








                • 3





                  Scientific Word comprises a TeX distribution along with a quasi-WYSIWYG front end. The code it generates is (mostly) LaTeX. I'm thus doubtful that Scientific Word can be considered a alternative to LaTeX. Separately, the software hasn't seen an upgrade in years and years. The current version, numbered 5.5, has been around since at least 2007 (and probably even longer). A software package that's been around essentially unchanged for more than seven years is functionally obsolete, I'm afraid.

                  – Mico
                  Jan 11 '15 at 17:22











                • @Mico As far as I agree, the same thing can essentially be said about TeX.

                  – Gaussler
                  Jul 11 '15 at 14:27











                • @Gaussler - Version 6 of Scientific Word has been released quite recently; I'm sure the long-suffering users of SW were jubilant. I don't agree that SW's situation is comparable to that of TeX. (i) TeXLive has been every year for many years now. (ii) While "Plain TeX" is frozen, that's not the case for pdfLaTeX, XeTeX, LuaTeX, and lots and lots of packages for pdfLaTeX, XeLaTeX, and LuaTeX. (iii) LaTeX2e was also nearly frozen for many years, but got a significant update as of 1/1/2015.

                  – Mico
                  Jul 11 '15 at 14:54













                • @Mico Still, the pdfTeX backend has remained dominant and frozen since forever. Also, you compare Plain TeX - a format - to LuaTeX and XeTeX - two engines.

                  – Gaussler
                  Jul 11 '15 at 15:01








                • 1





                  @Gaussler - I'm no good at linguistic quibbles. Can't vouch for what "standard" terminology may be. All I can hope for is that I've clarified what I meant to say.

                  – Mico
                  Jul 11 '15 at 15:25
















                • 3





                  Scientific Word comprises a TeX distribution along with a quasi-WYSIWYG front end. The code it generates is (mostly) LaTeX. I'm thus doubtful that Scientific Word can be considered a alternative to LaTeX. Separately, the software hasn't seen an upgrade in years and years. The current version, numbered 5.5, has been around since at least 2007 (and probably even longer). A software package that's been around essentially unchanged for more than seven years is functionally obsolete, I'm afraid.

                  – Mico
                  Jan 11 '15 at 17:22











                • @Mico As far as I agree, the same thing can essentially be said about TeX.

                  – Gaussler
                  Jul 11 '15 at 14:27











                • @Gaussler - Version 6 of Scientific Word has been released quite recently; I'm sure the long-suffering users of SW were jubilant. I don't agree that SW's situation is comparable to that of TeX. (i) TeXLive has been every year for many years now. (ii) While "Plain TeX" is frozen, that's not the case for pdfLaTeX, XeTeX, LuaTeX, and lots and lots of packages for pdfLaTeX, XeLaTeX, and LuaTeX. (iii) LaTeX2e was also nearly frozen for many years, but got a significant update as of 1/1/2015.

                  – Mico
                  Jul 11 '15 at 14:54













                • @Mico Still, the pdfTeX backend has remained dominant and frozen since forever. Also, you compare Plain TeX - a format - to LuaTeX and XeTeX - two engines.

                  – Gaussler
                  Jul 11 '15 at 15:01








                • 1





                  @Gaussler - I'm no good at linguistic quibbles. Can't vouch for what "standard" terminology may be. All I can hope for is that I've clarified what I meant to say.

                  – Mico
                  Jul 11 '15 at 15:25










                3




                3





                Scientific Word comprises a TeX distribution along with a quasi-WYSIWYG front end. The code it generates is (mostly) LaTeX. I'm thus doubtful that Scientific Word can be considered a alternative to LaTeX. Separately, the software hasn't seen an upgrade in years and years. The current version, numbered 5.5, has been around since at least 2007 (and probably even longer). A software package that's been around essentially unchanged for more than seven years is functionally obsolete, I'm afraid.

                – Mico
                Jan 11 '15 at 17:22





                Scientific Word comprises a TeX distribution along with a quasi-WYSIWYG front end. The code it generates is (mostly) LaTeX. I'm thus doubtful that Scientific Word can be considered a alternative to LaTeX. Separately, the software hasn't seen an upgrade in years and years. The current version, numbered 5.5, has been around since at least 2007 (and probably even longer). A software package that's been around essentially unchanged for more than seven years is functionally obsolete, I'm afraid.

                – Mico
                Jan 11 '15 at 17:22













                @Mico As far as I agree, the same thing can essentially be said about TeX.

                – Gaussler
                Jul 11 '15 at 14:27





                @Mico As far as I agree, the same thing can essentially be said about TeX.

                – Gaussler
                Jul 11 '15 at 14:27













                @Gaussler - Version 6 of Scientific Word has been released quite recently; I'm sure the long-suffering users of SW were jubilant. I don't agree that SW's situation is comparable to that of TeX. (i) TeXLive has been every year for many years now. (ii) While "Plain TeX" is frozen, that's not the case for pdfLaTeX, XeTeX, LuaTeX, and lots and lots of packages for pdfLaTeX, XeLaTeX, and LuaTeX. (iii) LaTeX2e was also nearly frozen for many years, but got a significant update as of 1/1/2015.

                – Mico
                Jul 11 '15 at 14:54







                @Gaussler - Version 6 of Scientific Word has been released quite recently; I'm sure the long-suffering users of SW were jubilant. I don't agree that SW's situation is comparable to that of TeX. (i) TeXLive has been every year for many years now. (ii) While "Plain TeX" is frozen, that's not the case for pdfLaTeX, XeTeX, LuaTeX, and lots and lots of packages for pdfLaTeX, XeLaTeX, and LuaTeX. (iii) LaTeX2e was also nearly frozen for many years, but got a significant update as of 1/1/2015.

                – Mico
                Jul 11 '15 at 14:54















                @Mico Still, the pdfTeX backend has remained dominant and frozen since forever. Also, you compare Plain TeX - a format - to LuaTeX and XeTeX - two engines.

                – Gaussler
                Jul 11 '15 at 15:01







                @Mico Still, the pdfTeX backend has remained dominant and frozen since forever. Also, you compare Plain TeX - a format - to LuaTeX and XeTeX - two engines.

                – Gaussler
                Jul 11 '15 at 15:01






                1




                1





                @Gaussler - I'm no good at linguistic quibbles. Can't vouch for what "standard" terminology may be. All I can hope for is that I've clarified what I meant to say.

                – Mico
                Jul 11 '15 at 15:25







                @Gaussler - I'm no good at linguistic quibbles. Can't vouch for what "standard" terminology may be. All I can hope for is that I've clarified what I meant to say.

                – Mico
                Jul 11 '15 at 15:25













                1














                Pollen, written in Racket (of the Lisp family). The primary focus is web-based typography (and no math so far), but it can also support other formats such as pdf.






                share|improve this answer




























                  1














                  Pollen, written in Racket (of the Lisp family). The primary focus is web-based typography (and no math so far), but it can also support other formats such as pdf.






                  share|improve this answer


























                    1












                    1








                    1







                    Pollen, written in Racket (of the Lisp family). The primary focus is web-based typography (and no math so far), but it can also support other formats such as pdf.






                    share|improve this answer













                    Pollen, written in Racket (of the Lisp family). The primary focus is web-based typography (and no math so far), but it can also support other formats such as pdf.







                    share|improve this answer












                    share|improve this answer



                    share|improve this answer










                    answered Apr 6 '18 at 23:36









                    user159867user159867

                    413




                    413























                        0














                        R Markdown runs on the TeX distribution when using kntr to create PDFs. But, to type in it requires less code memorization since R produces default YMAL headers when you load up a new document.






                        share|improve this answer








                        New contributor




                        Jennifer is a new contributor to this site. Take care in asking for clarification, commenting, and answering.
                        Check out our Code of Conduct.

























                          0














                          R Markdown runs on the TeX distribution when using kntr to create PDFs. But, to type in it requires less code memorization since R produces default YMAL headers when you load up a new document.






                          share|improve this answer








                          New contributor




                          Jennifer is a new contributor to this site. Take care in asking for clarification, commenting, and answering.
                          Check out our Code of Conduct.























                            0












                            0








                            0







                            R Markdown runs on the TeX distribution when using kntr to create PDFs. But, to type in it requires less code memorization since R produces default YMAL headers when you load up a new document.






                            share|improve this answer








                            New contributor




                            Jennifer is a new contributor to this site. Take care in asking for clarification, commenting, and answering.
                            Check out our Code of Conduct.










                            R Markdown runs on the TeX distribution when using kntr to create PDFs. But, to type in it requires less code memorization since R produces default YMAL headers when you load up a new document.







                            share|improve this answer








                            New contributor




                            Jennifer is a new contributor to this site. Take care in asking for clarification, commenting, and answering.
                            Check out our Code of Conduct.









                            share|improve this answer



                            share|improve this answer






                            New contributor




                            Jennifer is a new contributor to this site. Take care in asking for clarification, commenting, and answering.
                            Check out our Code of Conduct.









                            answered 39 mins ago









                            JenniferJennifer

                            907




                            907




                            New contributor




                            Jennifer is a new contributor to this site. Take care in asking for clarification, commenting, and answering.
                            Check out our Code of Conduct.





                            New contributor





                            Jennifer is a new contributor to this site. Take care in asking for clarification, commenting, and answering.
                            Check out our Code of Conduct.






                            Jennifer is a new contributor to this site. Take care in asking for clarification, commenting, and answering.
                            Check out our Code of Conduct.

















                                protected by yo' Aug 19 '14 at 14:03



                                Thank you for your interest in this question.
                                Because it has attracted low-quality or spam answers that had to be removed, posting an answer now requires 10 reputation on this site (the association bonus does not count).



                                Would you like to answer one of these unanswered questions instead?



                                Popular posts from this blog

                                Paper upload error, “Upload failed: The top margin is 0.715 in on page 3, which is below the required...

                                Emraan Hashmi Filmografia | Linki zewnętrzne | Menu nawigacyjneGulshan GroverGulshan...

                                How can I write this formula?newline and italics added with leqWhy does widehat behave differently if I...